Sie sind auf Seite 1von 40

Bataan Shipyard Engineering Co., Inc. vs. PCGG (G.R. No.

75885 May 27, 1987) is presumed to be incorporated for the benefit of the public. It received certain special
privileges and franchises, and holds them subject to the laws of the state and the
Facts: Challenged in this special civil action of certiorari and prohibition by a private limitations of its charter. It’s powers are limited by law. It can make no contract not
corporation known as the Bataan Shipyard and Engineering Co., Inc. are: (1) Executive authorized by its charter. Its rights to act as a corporation are only preserved to it so long
Orders Numbered 1 and 2, promulgated by President Corazon C. Aquino on February 28, as it obeys the laws of its creation. There is a reserve right in the legislature to investigate
1986 and March 12, 1986, respectively, and (2) the sequestration, takeover, and other its contracts and find out whether it has exceeded its powers. It would be a strange
orders issued, and acts done, in accordance with said executive orders by the Presidential anomaly to hold that a state, having chartered a corporation to make use of certain
Commission on Good Government and/or its Commissioners and agents, affecting said franchises, could not, in the exercise of sovereignty, inquire how these franchises had been
corporation. The sequestration order issued on April 14, 1986 was addressed to three of employed, and whether they had been abused, and demand the production of the
the agents of the Commission, ordering them to sequester several companies among corporate books and papers for that purpose. The defense amounts to this, that an officer
which is Bataan Shipyard and Engineering Co., Inc. On the strength of the above of the corporation which is charged with a criminal violation of the statute may plead the
sequestration order, several letters were sent to BASECO among which is that from Mr. criminality of such corporation as a refusal to produce its books. To state this proposition is
Jose M. Balde, acting for the PCGG, addressed a letter dated April 18, 1986 to the President to answer it. While an individual may lawfully refuse to answer incriminating questions
and other officers of petitioner firm, reiterating an earlier request for the production of unless protected by an immunity statute, it does not follow that a corporation, vested with
certain documents. The letter closed with the warning that if the documents were not special privileges and franchises may refuse to show its hand when charged with an abuse
submitted within five days, the officers would be cited for "contempt in pursuance with of such privileges. (Wilson v. United States, 55 Law Ed., 771, 780 [emphasis, the Solicitor
Presidential Executive Order Nos. 1 and 2." BASECO contends that its right against self General's]) The constitutional safeguard against unreasonable searches and seizures finds
incrimination and unreasonable searches and seizures had been transgressed by the Order no application to the case at bar either. There has been no search undertaken by any agent
of April 18, 1986 which required it "to produce corporate records from 1973 to 1986 under or representative of the PCGG, and of course no seizure on the occasion thereof.
pain of contempt of the Commission if it fails to do so." BASECO prays that the Court 1)
declare unconstitutional and void Executive Orders Numbered 1 and 2; 2) annul the Firme vs Bukal Enterprises and Dev. Corp
sequestration order dated April- 14, 1986, and all other orders subsequently issued and
acts done on the basis thereof, inclusive of the takeover order of July 14, 1986 and the 414 SCRA 190 (2003)
termination of the services of the BASECO executives.

Issue: Whether or not BASECO’s right against self-incrimination and unreasonable searches
and seizures was violated. This is a petition for review on certiorari of the Decision dated 3 January 2001 of the Court
of Appeals in CA-G.R. CV No. 60747. The Court of Appeals reversed the Decision of the
Regional Trial Court, Branch 223, Quezon City
Ruling: No. The order to produce documents was issued upon the authority of Section 3 (e)
of Executive Order No. 1, treating of the PCGG's power to "issue subpoenas requiring * *
the production of such books, papers, contracts, records, statements of accounts and other
documents as may be material to the investigation conducted by the Commission. It is Facts : Petitioner Spouses Constante and Azucena Firme ("Spouses Firme") are the
elementary that the right against self-incrimination has no application to juridical persons. registered owners of a parcel of land ("Property") located on Dahlia Avenue, Fairview Park,
While an individual may lawfully refuse to answer incriminating questions unless protected Quezon City. Renato de Castro ("De Castro"), the vice president of Bukal Enterprises and
by an immunity statute, it does not follow that a corporation, vested with special privileges Development Corporation ("Bukal Enterprises") authorized his friend, Teodoro Aviles
and franchises, may refuse to show its hand when charged with an abuse of such ("Aviles"), a broker, to negotiate with the Spouses Firme for the purchase of the Property.
privileges. Corporations are not entitled to all of the constitutional protections, which
private individuals have. They are not at all within the privilege against self-incrimination; Bukal Enterprises filed a complaint for specific performance and damages with the trial
although this court more than once has said that the privilege runs very closely with the court and asked the trial court to order the Spouses Firme to execute the deed of sale and
4th Amendment's Search and Seizure provisions. It is also settled that an officer of the to deliver the title to the Property to Bukal Enterprises upon payment of the agreed
company cannot refuse to produce its records in its possession upon the plea that they will purchase price.
either incriminate him or may incriminate it." The corporation is a creature of the state. It
Aviles , one of the witnesses, testified that he was authorized to represent Bukal Ruling: The Supreme Court ruled that there is no perfected contract of sale. Records
Enterprises and he presented a draft of the Deed of Sale to the petitioners but such draft is indubitably show that there was no consent on the part of the Spouses Firme. Spouses
rejected due to several objectionable conditions, including the payment of capital gains Firme found the terms and conditions unacceptable and told Aviles that they would not
and other government taxes by the seller and the relocation of the squatters at the seller’s sell the property. De Castro also admitted that he was aware of the Spouses Firme’s refusal
expense. Allegedly the petitioners accepted the second draft upon the deletion of the to sell the Property. The confusing testimony of Aviles taken together with De Castro’s
objectionable conditions and agreed that payment would be made at the Far East Bank admission that he was aware of the Spouses Firme’s refusal to sell the Property reinforces
and Trust Company ("FEBTC"), Padre Faura Branch, Manila. However, the scheduled Dr. Firme’s testimony that he and his wife never consented to sell the Property. The
payment had to be postponed due to problems in the transfer of funds and after that the essence of consent is the conformity of the parties on the terms of the contract, the
spouses informed Aviles that they were no longer interested in selling the Property.Bukal acceptance by one of the offer made by the other. The contract to sell is a bilateral
Enterprises then filed a complaint for specific performance and damages. contract. Where there is merely an offer by one party, without the acceptance of the
other, there is no consent. Assuming there is a valid sale, there was no approval from the
On the other hand, Dr. Constante Firme ("Dr. Firme") was the sole witness for the Board of Directors of Bukal Enterprises as would finalize any transaction with the Spouses
defendant, testified that on 30 January 1995, he and his wife met with Aviles at the Firme. Aviles did not have the proper authority to negotiate for Bukal Enterprises. Aviles
Aristocrat Restaurant in Quezon City. Aviles arranged the meeting with the Spouses Firme testified that his friend, De Castro, had asked him to negotiate with the Spouses Firme to
involving their Property in Fairview. Aviles offered to buy the Property at P2,500 per square buy the Property. However, there is no Board Resolution authorizing Aviles to negotiate
meter. The Spouses Firme did not accept the offer because they were reserving the and purchase the Property on behalf of Bukal Enterprises. It is the board of directors or
Property for their children. On 6 February 1995, the Spouses Firme met again with Aviles trustees which exercises almost all the corporate powers in a corporation. The Corporation
upon the latter’s insistence. Aviles showed the Spouses Firme a copy of a draft deed of sale Code provides :
("Third Draft") which Aviles prepared.

Spouses Firme did not accept the Third Draft because they found its provisions one-sided.
The Spouses Firme particularly opposed the provision on the delivery of the Property’s title SEC. 23. The board of directors or trustees. — Unless otherwise provided in this Code, the
to Bukal Enterprises for the latter to obtain a loan from the bank and use the proceeds to corporate powers of all corporations formed under this Code shall be exercised, all
pay for the Property. The Spouses Firme repeatedly told Aviles that the Property was not business conducted and all property of such corporations controlled and held by the board
for sale when Aviles called on 2 and 4 March 1995 regarding the Property. Upon visit in of directors or trustees to be elected from among the holders of stock, or where there is
their property, the spouses saw that there are already improvements made and the no stock, from among the members of the corporation, who shall hold office for one (1)
squatters vacated the premises. year and until their successors are elected and qualified.

On 22 March 1995, the Spouses Firme received a letter dated 7 March 1995 from Bukal
Enterprises demanding that they sell the Property and on 7 August 1998, the trial court
rendered judgment against Bukal Enterprises dissmissing the complaint. SEC. 36. Corporate powers and capacity. — Every corporation incorporated under this
Code has the power and capacity:
Bukal Enterprises appealed to the Court of Appeals, which reversed and set aside the
decision of the trial court. xxx

To purchase, receive, take or grant, hold, convey, sell, lease, pledge, mortgage and
otherwise deal with such real and personal property, including securities and bonds of
Issue: Whether there is a perfected sale?if so is it valid despite there is a lack of other corporations, as the transaction of a lawful business of the corporation may
authorization from the Board of Directors? reasonably and necessarily require, subject to the limitations prescribed by the law and the
Constitution.

Under these provisions, the power to purchase real property is vested in the board of
directors or trustees. While a corporation may appoint agents to negotiate for the
purchase of real property needed by the corporation, the final say will have to be with the concurred in by the Commission en banc are fully supported by the evidence on record
board, whose approval will finalize the transaction. A corporation can only exercise its which clearly points out that R. F. Sugay & Co., is the statutory employer of the claimants.
powers and transact its business through its board of directors and through its officers and The decisive elements showing that it is the employer, are present, such as selection and
agents when authorized by a board resolution or its by-laws. engagement; payment of wages; power of dismissal, and control.

There was a faint attempt by the petitioning corporation, to evade liability, by advancing
the theory that Romulo P. Sugay, its President, was the one who entered into a contract of
R.F. Sugay vs Reyes 12 SCRA 700 (1964) administration and supervision for the painting of the factory of the Pacific Products, Inc.,
and making it appear that said Romulo F. Sugay acted as an agent of the Pacific Products,
An appeal from a decision of the Workmen's Compensation Commission Inc., and as such, the latter should be made answerable to the compensation due to the
claimants. We, however, agree with the Commission that "the dual roles of Romulo F.
Facts : Respondents Pablo Reyes and Cesar Curata suffered burns of various degrees, while
Sugay should not be allowed to confuse the facts relating to employer-employee
painting the building of the Pacific Products, Inc., caused by a fire of accidental origin,
relationship." It is a legal truism that when the veil of corporate fiction is made as a shield
resulting in their temporary disability from work. For said injuries they filed claims for
to perpetrate a fraud and/or confuse legitimate issues (here, the relation of employer-
disability and medical expenses against the R. F. Sugay & Co., Inc., Romulo F. Sugay and the
employee), the same should be pierced. Verily the R. F. Sugay & Co., Inc. is a business
Pacific Products, Inc. The R. F. Sugay & Co., Inc., answered the claim, alleging that the
conduit of R. F. Sugay.
corporation was not the employer of the claimants but it was the Pacific Products, Inc.,
which had an administration and supervision job contract with Romulo F. Sugay, who,
aside from being the President of the corporation, bearing his name, had also a business of
his own, distinct and separate from said corporation; and that the Regional Office of the
Department of Labor had no jurisdiction over the subject matter. Romulo Sugay voluntary
appeared during the scheduled hearings and denied the liabilities. Pacific Products, Inc. on
the other hand averred that its business was mainly in the manufacture and sale of lacquer
and other painting materials. As defenses, it stated that the claimants were the employees
of respondents R. F. Sugay Construction Co., Inc., and/or Romulo F. Sugay. The Hearing
Officer dismissed the case and exempted R. F. Sugay Construction Co., Inc., and Romulo F.
Sugay from any liability for lack of employer-employee relationship with the claimants. . RICARDO TANTONGCO, petitioner, v. KAISAHAN NG MGA MANGGAGAWA SA LA
The officer ordered Pacific Products to pay the injured workers. Pacific Products, Inc., CAMPANA (KKM) and THE HONORABLE COURT OF INDUSTRIAL RELATIONS, respondents
appealed the above decision to the Commission and Commissioner Jose Sanchez rendered
judgment affirming the compensability of the injuries and the amounts due them, but GR No. L-13119 || September 22, 1959
modified the decision of the Hearing Officer, by finding that R. F. Sugay & Co., Inc., was the
statutory employer of the claimants and should be liable to them. Pacific Products, Inc.,
was absolved from all responsibility. R. F. Sugay Construction Co., Inc. filed a motion of
reconsideration but the Commission en banc denied the motion. The present case is a petition for Certiorari and prohibition with prayer for the issuance of
a writ of preliminary injunction to prohibit the respondent Court of Industrial Relations
from proceeding with the hearing of the contempt proceedings.

Issue : Is R.F. Sugay construction Co., Inc. the employer of the injured workers? Is it liable?

FACTS La Campana Starch Factory and La Campana Coffee Factory (La Campana for
Brevity) are two separate entities run by a single management under the leadership of
Ruling: The Supreme Court ruled that R.F. Sugay construction Co., Inc. is the employer of Ramon Tantongco. Kaisahan ng mga Manggagawa sa La Campana (Kaisahan for brevity), on
the workers. The Court find that the findings of facts made by the Commissioner and the other hand, is a labor union with members from the two companies. Sometime in June,
1951, representatives of Kaisahan approached the management of La Campana to demand 3. WON the contempt of court proceedings in the CIR should proceed.
higher wages and more benefits. A deadlock ensued since none of the parties is willing to
give concessions. The dispute was certified to the Court of Industrial Relations (CIR). La
Campana filed a motion to dismiss before the CIR claiming that the CIR has no jurisdiction
because only those from the coffee factory were presenting the demands there were only RULING The Supreme Court DENIED the Petition for Certiorari and Prohibition. It ruled that
14 employees in said factory. This was done in light of the requirement that at least 31 La Camapana continued to exist despite the death of Ramon Tantongco. It further ruled
employees should present the demands. The motion was denied by the CIR. According to that the Doctrine of Piercing the Veil of Corporate Existence is not applicable in the present
the CIR, the Kaisahan was the one that presented the demands and not just the workers in case. Finally, it allowed the CIR to proceed with the contempt hearing.
the coffee factory. The Supreme Court affirmed the order of the CIR citing that although
the two entities are separate, there is only one management. The entire membership of
the Kaisahan is therefore to be counted and not simply those employed in the coffee
1 and 2
factory. Additional incidental cases were filed by Kaisahan before the CIR including a
petition for the reinstatement of some employees. Ramon Tantongco died some time in
The death of Ramon Tantongco did not end the existence of La Campana. The Supreme
1956. The administrator of the estate of Ramon Tantongco, herein petitioner Ricardo
Court applied the Doctrine of Piercing the Veil of Corporate Existence in GR no. L-5677 to
Tantongco, was ordered included as respondent in the cases pending before the CIR. The
avoid the use of technicality to defeat the jurisdiction of the CIR. In the said case, the Court
CIR rendered a decision on the incidental cases and ordered the reinstatement of the
determined that although La Campana are two separate companies, they are being
dismissed employees. When the employees reported to work, the management refused
managed by only one management. Furthermore, the workers of both factories were
them admittance. Kaisahan then filed a petition to cite the management in contempt
interchangeably assigned. In the present case, however, the Court ruled that despite the
before the CIR. Hence this petition.
obvious fact that La Campana was run by the same people, they still are two different
companies with separate personalities from Ramon Tantongco. La Campana was owned
not only by Ramon but others as well including Ricardo Tantongco. Lastly, the Court ruled
CONTENTIONS Petitioner: The two companies ceased to exist upon the death of Ramon that petitioner is under estoppel and cannot claim that La Campana and Ramon are one
Tantongco. The Supreme Court held in GR No. L-5677 that La Campana and Ramon and the same since he has represented La Campana as separate entities in numerous
Tantongco are one based on the doctrine of piercing the veil of corporate existence. dealings.
Therefore, the death of Ramon Tantongco meant the death of La Campana. Since La
Campana already ceased to exist, the CIR no longer has jurisdiction over it. The claims
should have been filed with the probate court.
3. Ricardo Tantongco should still face the contempt proceedings because under Section 6
of Commonwealth Act No. 143, “In case the employer (or landlord) committing any such
Defendant: La Campana continues to exist despite the death of Ramon Tantongco. The CIR
violation or contempt is an association or corporation, the manager or the person who has
therefore has jurisdiction when it rendered its decision on the incidental cases. The non-
the charge of the management of the business of the association or corporation and the
compliance by La Campana therefore amounted to contempt of court.
officers of directors thereof who have ordered or authorized the violation of contempt
shall be liable. . . .” Since Tantongco is the General Manager of La Campana, he is still
obliged to appear at the contempt proceedings.
ISSUE

1. WON La Campana ceased to exist upon the death of Ramon Tantongco;


G.R. No. L-67626 April 18, 1989
2. WON the Doctrine of Piercing the Veil of Corporate Existence applies to the present case;
JOSE REMO, JR., petitioner, vs. THE HON. INTERMEDIATE APPELLATE COURT and E.B.
and
MARCHA TRANSPORT COMPANY, INC., represented by APIFANIO B. MARCHA,
respondents.
application from a certain financing company, and that ten (10) trucks have been returned
to Bagbag, Novaliches.
Nature: Petition for review, seeking the reversal of the decision of the Intermediate
Appellate Court In due time, private respondent filed a compliant for the recovery of P525,000.00 or the
return of the 13 trucks with damages against Akron and its officers and directors with the
then Court of First Instance of Rizal. Only petitioner answered the complaint denying any
participation in the transaction and alleging that Akron has a distinct corporate personality.
Facts:On December, 1977, the board of directors of Akron Customs Brokerage Corporation, He was, however, declared in default for his failure to attend the pre-trial. Petitioner on
of which petitioner Jose Remo, Jr. was a member, adopted a resolution authorizing the the other hand, sold all his shares in Akron to Copranda. Akron thenafter changed its name
purchase of thirteen (13) trucks for use in its business to be paid out of a loan the to Akron Transport International, Inc.
corporation may secure from any lending institution.
The trial court ruled in favor of private respondents, ordering petitioner to pay the
purchase price for the 13 trucks, rentals, attorney's fees and the cost of suit. On appeal,
the IAC reversed the decision of the trial court. However, on motion for reconsideration,
Feliciano Coprada, as President and Chairman of Akron, purchased the 13 trucks from
the IAC affirmed the appealed decision of the CFI.
private respondent for P525,000.00 as evidenced by a deed of absolute sale. In a side
agreement of the same date, the parties agreed on a downpayment of P50,000.00 and that Issue: Was the IAC correct in disregarding corporate fiction and holding petitioner
the balance of P475,000.00 to be paid within sixty (60) days from the date of the execution personally liable for the obligation of the Corporation?
of the agreement. The parties also agreed that until said balance is fully paid, the down
payment of P50,000.00 shall accrue as rentals of the 13 trucks; and failure of Akron to pay Was the IAC correct in sanctioning the merger of the personality of the corporation with
the balance within the period of 60 days shall create a chattel mortgage lien covering said that of the petitioner when the latter was held liable for the corporate debts?
cargo trucks and the parties may allow an extension of 30 days and thereafter private
respondent may ask for a revocation of the contract and the reconveyance of all said Petitioner's contention: Akron has a distinct corporate personality. As such, he cannot be
trucks. The obligation is secured by a promissory note executed by Coprada in favor of held personally liable for the liabilities of the corporation.
Akron. It is stated in the promissory note that the balance shall be paid from the proceeds
of a loan obtained from the Development Bank of the Philippines (DBP) within sixty (60) Private respondent's contention: It is a victim of fraud, which merits the piercing of
days. corporate fiction

After the lapse of 90 days, private respondent tried to collect from Coprada but the latter Held: Petitioner cannot be held personally liable. There is no basis to pierce the corporate
promised to pay only upon the release of the DBP loan. Private respondent sent Coprada a veil of Akron and hold petitioner personally liable for its obligation to private respondent.
letter of demand dated May 10, 1978. In his reply to the said letter, Coprada reiterated While it is true that petitioner was still a member of the board of directors of Akron when a
that he was applying for a loan from the DBP from the proceeds of which payment of the resolution was adopted authorizing the purchase of 13 trucks, it does not appear that said
obligation shall be made. resolution was intended to defraud anyone and more particularly private respondent.

Upon inquiry, private respondent found that no loan application was ever filed by Akron It was Coprada, President and Chairman of Akron, who negotiated with said respondent for
with DBP. Coprada wrote private respondent begging for a grace period of until the end of the purchase of 13 cargo trucks, who signed a promissory note to guarantee the payment
the month to pay the balance of the purchase price, promising that he will update the of the unpaid balance of the purchase price out of the proceeds of a loan he supposedly
rentals within the week; and in case he fails, then he will return the 13 units should private sought from the DBP. The word "WE' in the said promissory note must refer to the
respondent elect to get back the same. Private respondent, through counsel, wrote Akron corporation which Coprada represented in the execution of the note and not its
on August 1, 1978 demanding the return of the 13 trucks and the payment of P25,000.00 stockholders or directors. Petitioner did not sign the said promissory note so he cannot be
back rentals covering the period from June 1 to August 1, 1978. Coprada again asked for personally bound thereby. It is Coprada who should account for the same and not
another grace period stating as well that he is expecting the approval of his loan petitioner.
As to the amendment of the articles of incorporation of Akron thereby changing its name January 27, 1997: National Investment and Development Corporation (NIDC), a
to Akron Transport International, Inc., petitioner alleges that the change of corporate government corporation, entered into a Joint Venture Agreement (JVA) with Kawasaki
name was in order to include trucking and container yard operations in its customs Heavy Industries, Ltd. of Kobe, Japan (KAWASAKI) for the construction, operation and
brokerage of which private respondent was duly informed in a letter. 19 Indeed, the new management of the Subic National Shipyard, Inc. (SNS) which subsequently became the
corporation confirmed and assumed the obligation of the old corporation. There is no Philippine Shipyard and Engineering Corporation (PHILSECO). Under the JVA, the NIDC and
indication of an attempt on the part of Akron to evade payment of its obligation to private KAWASAKI will contribute P330 million for the capitalization of PHILSECO in the proportion
respondent. of 60%-40% respectively. One of its salient features is the grant to the parties of the right
of first refusal should either of them decide to sell, assign or transfer its interest in the
There is the fact that petitioner sold his shares in Akron to Coprada during the pendency of joint venture.
the case. Since petitioner has no personal obligation to private respondent, it is his
inherent right as a stockholder to dispose of his shares of stock anytime he so desires. November 25, 1986: NIDC transferred all its rights, title and interest in PHILSECO to the
PNB, interests transferred to the National Government pursuant to Admin Order No. 14.
On December 8, 1986, Pres. Corazon C. Aquino issued Proclamation No. 50 establishing the
Committee on Privatization (COP) and the Asset Privatization Trust (APT) to take title to,
Ruling: WHEREFORE, the petition is GRANTED. The questioned resolution of the and possession of, conserve, manage and dispose of non-performing assets of the National
Intermediate Appellate Court dated February 8,1984 is hereby set aside and its decision Government. Thereafter, on February 27, 1987, a trust agreement was entered into
dated June 30,1983 setting aside the decision of the trial court dated October 28, 1980 between the National Government and the APT wherein the latter was named the trustee
insofar as petitioner is concemed is hereby reinstated and affirmed, without costs. of the National Government's share in PHILSECO. In 1989, as a result of a quasi-
reorganization of PHILSECO to settle its huge obligations to PNB, the National
Government's shareholdings in PHILSECO increased to 97.41% thereby reducing
KAWASAKI's shareholdings to 2.59%.

In the interest of the national economy and the government, the COP and the APT deemed
it best to sell the National Government's 87.6% share in PHILSECO (896,869,942
outstanding capital stock) to private entities in the Indicative Price Bidding Basis of
P1,300,000,000.00. After a series of negotiations between the APT and KAWASAKI, they
G.R. No. 124293 January 31, 2005
agreed that the latter's right of first refusal under the JVA be "exchanged" for the right to
J.G. SUMMIT HOLDINGS, INC., petitioner, top by 5% the highest bid for the said shares. They further agreed that KAWASAKI would
be entitled to name a company in which it was a stockholder, which could exercise the
vs. right to top. On September 7, 1990, KAWASAKI informed APT that PHILYARDS Holdings,
Inc. (PHI) would exercise its right to top. A pre-bidding conference was held on September
COURT OF APPEALS; COMMITTEE ON PRIVATIZATION, its Chairman and Members; ASSET 18, 1993. The highest bid, as well as the buyer, will be subject to final approval of both APT
PRIVATIZATION TRUST; and PHILYARDS HOLDINGS, INC., respondents. and COP, and APT reserves the right in its sole discretion to reject any or all bids.

RESOLUTION xxx xxx xxx

The APT shall advise Kawasaki Heavy Industries, Inc. and/or its nominee, [PHILYARDS]
Holdings, Inc., that the highest bid is acceptable to the National Government. Kawasaki
PUNO, J.: Heavy Industries, Inc. and/or [PHILYARDS] Holdings, Inc. shall then have a period of thirty
(30) calendar days from the date of receipt of such advice from APT within which to
I. Facts exercise their "Option to Top the Highest Bid" by offering a bid equivalent to the highest
bid plus five (5%) percent thereof.
6.1 Should Kawasaki Heavy Industries, Inc. and/or [PHILYARDS] Holdings, Inc. exercise their PHILSECO to Philyard[s] Holdings, Inc., and upheld the right of JG Summit, as the highest
"Option to Top the Highest Bid," they shall so notify the APT about such exercise of their bidder, to take title to the said shares, viz:
option and deposit with APT the amount equivalent to ten percent (10%) of the highest bid
plus five percent (5%) thereof within the thirty (30)-day period mentioned in paragraph 6.0 (a) accept the said amount of P2,030,000,000.00 less bid deposit and interests from
above. APT will then serve notice upon Kawasaki Heavy Industries, Inc. and/or [PHILYARDS] petitioner;
Holdings, Inc. declaring them as the preferred bidder and they shall have a period of ninety
(90) days from the receipt of the APT's notice within which to pay the balance of their bid (b) execute a Stock Purchase Agreement with petitioner;
price.
(c) cause the issuance in favor of petitioner of the certificates of stocks representing 87.6%
6.2 Should Kawasaki Heavy Industries, Inc. and/or [PHILYARDS] Holdings, Inc. fail to of PHILSECO's total capitalization;
exercise their "Option to Top the Highest Bid" within the thirty (30)-day period, APT will
(d) return to private respondent PHGI the amount of Two Billion One Hundred Thirty-One
declare the highest bidder as the winning bidder.
Million Five Hundred Thousand Pesos (P2,131,500,000.00); and
xxx xxx xxx
(e) cause the cancellation of the stock certificates issued to PHI.
At the public bidding on the said date, petitioner J.G. Summit Holdings, Inc. submitted
SO ORDERED.
P2,030,000,000.00 bid with an acknowledgment of KAWASAKI/[PHILYARDS'] right to top.
xxx

As petitioner was declared the highest bidder, the COP approved the sale on December 3, II. Issues
1993 "subject to the right of Kawasaki Heavy Industries, Inc./[PHILYARDS] Holdings, Inc. to
top JGSMI's bid by 5% as specified in the bidding rules." Respondents submitted four basic issues for Resolution:

December 29, 1993: Petitioner informed APT that it was protesting the offer of PHI to top (1) Whether PHILSECO is a public utility;
its bid on the grounds that: (a) the KAWASAKI/PHI consortium composed of KAWASAKI,
[PHILYARDS], xx violated the ASBR because the last four (4) companies were the losing (2) Whether under the 1977 JVA, KAWASAKI can exercise its right of first refusal only up to
bidders thereby circumventing the law and prejudicing the weak winning bidder; 40% of the total capitalization of PHILSECO; and

(b) only KAWASAKI could exercise the right to top; (c) giving the same option to top to PHI (3) Whether the right to top granted to KAWASAKI violates the principles of competitive
constituted unwarranted benefit to a third party; (d) no right of first refusal can be bidding.
exercised in a public bidding or auction sale;
(4) that the maintenance of the 60%-40% relationship between the National Investment
February 2, 1994: Petitioner was notified that PHI had fully paid the balance of the and Development Corporation (NIDC) and KAWASAKI arises from contract and from the
purchase price of the subject bidding. On February 7, 1994, the APT notified petitioner that Constitution because PHILSECO is a landholding corporation and need not be a public
PHI had exercised its option to top the highest bid and that the COP had approved the utility to be bound by the 60%-40% constitutional limitation. Whether the exercise of its
same on January 6, 1994. On February 24, 1994, the APT and PHI executed a Stock right of first refusal by KAWASAKI of the 40% PHILSECO shares, a landholding corporation,
Purchase Agreement. violates Constitutional provisions on foreign equity ratio of 60%-40% and yet owns long-
term leasehold rights which are real rights and also continues to own real property.
November 20, 2000: this Court reversed CA’ ruling that shipyard (PHILSECO) is a public
utility whose capitalization must be 60% Filipino-owned. Consequently, the right to top
granted to KAWASAKI drafted for the sale of 87.67% equity of the National Government in
PHILSECO is illegal- because it allows foreign corporations to own more than 40% equity in III. Held
the shipyard. This Court voided the transfer of the national government's 87.67% share in
In a Resolution dated September 24, 2003, this Court ruled in favor of the respondents. "Save in cases of hereditary succession, no private lands shall be transferred or conveyed
except to individuals, corporations, or associations qualified to acquire or hold lands of the
(1) We held that Philippine Shipyard and Engineering Corporation (PHILSECO) is not a public domain."
public utility, as by nature, a shipyard is not a public utility and that no law declares a
shipyard to be a public utility. 32.1 This provision is the same as Section 7, Article XII of the 1987 Constitution.

(2) We found nothing in the 1977 Joint Venture Agreement (JVA) which prevents Kawasaki 32.2 Under the Public Land Act, corporations qualified to acquire or hold lands of the
Heavy Industries, Ltd. of Kobe, Japan (KAWASAKI) from acquiring more than 40% of public domain are corporations at least 60% of which is owned by Filipino citizens (Sec. 22,
PHILSECO’s total capitalization. Commonwealth Act 141, as amended)

(3) We held that the right to top granted to KAWASAKI in exchange for its right of first The prohibition in the Constitution applies only to ownership of land. It does not extend to
refusal did not violate the principles of competitive bidding. immovable or real property as defined under Article 415 of the Civil Code. Otherwise, we
would have a strange situation where the ownership of immovable property such as trees,
(4) No law disqualifies a person from purchasing shares in a landholding corporation even if plants and growing fruit attached to the land would be limited to Filipinos and Filipino
the latter will exceed the allowed foreign equity, what the law disqualifies is the corporations only.
corporation from owning land.
PHILSECO still owns land, the right of first refusal can be validly assigned to a qualified
Even if PHILYARDS owned land at the time of bidding, KAWASAKI had a valid right of first Filipino entity in order to maintain the 60%-40% ratio. This transfer, by itself, does not
refusal over PHILSECO shares under the JVA considering that PHILSECO owned land until amount to a violation of the Anti-Dummy Laws, absent proof of any fraudulent intent.
the time of the bidding and KAWASAKI can exceed 40% of PHILSECO’s equity, even it may Alternatively, In fact, it can even be said that if the foreign shareholdings of a landholding
have previously held land but divested such landholdings, or retained, the right of first corporation exceeds 40%, it is not the foreign stockholders’ ownership of the shares
refusal, being a property right, could be assigned to a qualified party. The mutual right of which is adversely affected but the capacity of the corporation to own land – that is, the
first refusal in favor of NIDC and KAWASAKI does not amount to a virtual transfer of land to corporation becomes disqualified to own land. This finds support under the basic
a non-Filipino. In fact, the case at bar involves a right of first refusal over shares of stock, corporate law principle that the corporation and its stockholders are separate juridical
not an option to buy the land itself. The transfer could be made either to a nominee or entities. In this vein, the right of first refusal over shares pertains to the shareholders
such other party which the holder of the right of first refusal feels it can comfortably do whereas the capacity to own land pertains to the corporation. Hence, the fact that
business with. As discussed earlier, there is a distinction between the shareholder’s PHILSECO owns land cannot deprive stockholders of their right of first refusal. This is the
ownership of shares and the corporation’s ownership of land arising from the separate clear import of the following provisions in the Constitution:
juridical personalities of the corporation and its shareholders.
Section 2. xxx The State may directly undertake such activities, or it may enter into co-
We uphold the validity of the mutual rights of first refusal under the JVA between production, joint venture, or production-sharing agreements with Filipino citizens, or
KAWASAKI and NIDC. First of all, the right of first refusal is a property right of PHILSECO corporations or associations at least sixty per centum of whose capital is owned by such
shareholders, KAWASAKI and NIDC, under the terms of their JVA. This right allows them to citizens. xxx
purchase the shares of their co-shareholder before they are offered to a third party. The
agreement of co-shareholders to mutually grant this right to each other, by itself, does xxx xxx xxx
not constitute a violation of the provisions of the Constitution limiting land ownership to
Filipinos and Filipino corporations. The petitioner further argues that "an option to buy land is void in itself. The right of first
refusal granted to KAWASAKI, a Japanese corporation, is similarly void. Hence, the right to
32. To review the constitutional provisions involved, Section 14, Article XIV of the 1973 top, sourced from the right of first refusal, is also void." The case of Lui She did not that say
Constitution (the JVA was signed in 1977), provided: "an option to buy land is void in itself," for it is held that Lease to an alien for a reasonable
period is valid, and an option giving an alien the right to buy real property is not completely
excluded by Constitution from the use of lands for residential purpose, and its temporary
residence may be given temporary rights such as lease contract which is not forbidden by vehicles to the public with a steep mark-up. Since then, Liddell & Co. paid sales taxes on
the same. the basis of its sales to Liddell Motors Inc. considering said sales as its original sales.

LIDDELL & CO., INC., petitioner-appellant, vs. THE COLLECTOR OF INTERNAL


REVENUE, respondent-appellee.
PETITIONER-APPELLANT: Petitioner filed an appeal on the decision of the Court of Tax
Appeals affirming the position taken by the Collector of Internal Revenue.
(G.R. No. L-9687, 30 June 1961)

RESPONDENT-APPELLEE: Upon review of the transactions between Liddell & Co. and Liddell
This is an appeal from the decision of the Court of Tax Appeals imposing a tax deficiency
Motors, Inc. the Collector of Internal Revenue determined that the latter was but an alter
liability on Liddell & Co., Inc.
ego of Liddell & Co. Wherefore, he concluded, that for sales tax purposes, those sales
made by Liddell Motors, Inc. to the public were considered as the original sales of Liddell &
Co. Accordingly, the Collector of Internal Revenue assessed against Liddell & Co. a sales tax
FACTS: The petitioner, Liddell & Co. Inc., (Liddell & Co. for short) is a domestic corporation deficiency, including surcharges. In the computation, the gross selling price of Liddell
establish in the Philippines on February 1, 1946, with an authorized capital of P100,000 Motors, Inc. to the general public from January 1, 1949 to September 15, 1950, was made
divided into 1000 share at P100 each. Of this authorized capital, 196 shares valued at the basis without deducting from the selling price, the taxes already paid by Liddell & Co. in
P19,600 were subscribed and paid by Frank Liddell while the other four shares were in the its sales to the Liddell Motors Inc.
name of Charles Kurz, E.J. Darras, Angel Manzano and Julian Serrano at one shares each. Its
purpose was to engage in the business of importing and retailing Oldsmobile and Chevrolet
passenger cars and GMC and Chevrolet trucks. After its incorporation, Lidell & Co. was able
ISSUE: Whether or not Liddell Motors, Inc. is the alter ego of Liddell & Co. Inc.?
to declare stock dividends, thereby increasing the issued capital stock of the said
corporation, which were duly approved by the Securities and Exchange Commission. There
has also been an agreement executed by Frank Lidell on one hand, and Messrs. Kurz,
Darras, Manzano and Serrano on the other, which was further supplemented by two other RULING: There are quite a series of conspicuous circumstances that militate against the
agreements wherein Frank Liddell transferred to various employees of Liddell & Co. shares separate and distinct personality of Liddell Motors, Inc. from Liddell & Co. We notice that
of stock. On the basis of the agreement, "40%" of the earnings available for dividends the bulk of the business of Liddell & Co. was channeled through Liddell Motors, Inc. On the
accrued to Frank Liddell although at the time of the execution of said instrument, Frank other hand, Liddell Motors, Inc. pursued no activities except to secure cars, trucks, and
Liddell owned all of the shares in said corporation. From 1946 until November 22, 1948, spare parts from Liddell & Co. Inc. and then sell them to the general public. These sales of
when the purpose clause of the Articles of Incorporation of Liddell & Co. Inc., was vehicles by Liddell & Co. to Liddell Motors, Inc. for the most part were shown to have taken
amended so as to limit its business activities to importations of automobiles and trucks, place on the same day that Liddell Motors, Inc. sold such vehicles to the public. We may
Liddell & Co. was engaged in business as an importer and at the same time retailer of even say that the cars and trucks merely touched the hands of Liddell Motors, Inc. as a
Oldsmobile and Chevrolet passenger cars and GMC and Chevrolet trucks. On December 20, matter of formality.
1948, the Liddell Motors, Inc. was organized and registered with the Securities and
Exchange Commission with an authorized capital stock of P100,000 of which P20,000 was It is of course accepted that the mere fact that one or more corporations are owned and
subscribed and paid for as follows: Irene Liddell wife of Frank Liddell 19,996 shares and controlled by a single stockholder is not of itself sufficient ground for disregarding separate
Messrs. Marcial P. Lichauco, E. K. Bromwell, V. E. del Rosario and Esmenia Silva, 1 share corporate entities. Authorities support the rule that it is lawful to obtain a corporation
each. At about the end of the year 1948, Messrs. Manzano, Kurz and Kernot resigned from charter, even with a single substantial stockholder, to engage in a specific activity, and such
their respective positions in the Retail Dept. of Liddell & Co. and they were taken in and activity may co-exist with other private activities of the stockholder. If the corporation is a
employed by Liddell Motors, Inc. Beginning January, 1949, Liddell & Co. stopped retailing substantial one, conducted lawfully and without fraud on another, its separate identity is
cars and trucks; it conveyed them instead to Liddell Motors, Inc. which in turn sold the to be respected. Accordingly, the mere fact that Liddell & Co. and Liddell Motors, Inc. are
corporations owned and controlled by Frank Liddell directly or indirectly is not by itself the tickets received, the Federation made two partial payments, both in September of
sufficient to justify the disregard of the separate corporate identity of one from the other. 1989, in the total amount of P176,467.50.
There is, however, in this instant case, a peculiar consequence of the organization and
activities of Liddell Motors, Inc.

Under the law in force at the time of its incorporation the sales tax on original sales of cars On 4 October 1989, petitioner wrote the Federation, through the private
(sections 184, 185 and 186 of the National Internal Revenue Code), was progressive, i.e. respondent a demand letter requesting for the amount of P265,894.33. On 30 October
10% of the selling price of the car if it did not exceed P5000, and 15% of the price if more 1989, the Federation, through the Project Gintong Alay, paid the amount of
than P5000 but not more than P7000, etc. This progressive rate of the sales tax naturally P31,603.00. On 27 December 1989, Henri Kahn issued a personal check in the amount of
would tempt the taxpayer to employ a way of reducing the price of the first sale. And P50,000 as partial payment for the outstanding balance. No further payments were made
Liddell Motors, Inc. was the medium created by Liddell & Co. to reduce the price and the despite repeated demands prompting the Travel Agency to file a civil case before the
tax liability. Regional Trial Court of Manila. The Travel Agency sued Henri Kahn in his personal capacity
and as President of the Federation and impleaded the Federation as an alternative
As opined in the case of Gregory v. Helvering, "the legal right of a taxpayer to decrease the defendant. The Travel Agency sought to hold Henri Kahn liable on the ground that he
amount of what otherwise would be his taxes, or altogether avoid them by means which allegedly guaranteed the said obligation.
the law permits, cannot be doubted." But, as held in another case, "where a corporation is
a dummy, is unreal or a sham and serves no business purpose and is intended only as a
blind, the corporate form may be ignored for the law cannot countenance a form that is
bald and a mischievous fiction." Consistently with this view, the United States Supreme While not denying the allegation that the Federation owed the unpaid balance in the
Court held that "a taxpayer may gain advantage of doing business thru a corporation if he amount of P207,524.20, Kahn averred that there was no cause of action against him either
pleases, but the revenue officers in proper cases, may disregard the separate corporate in his personal capacity or in his official capacity as president of the Federation. He
entity where it serves but as a shield for tax evasion and treat the person who actually may maintained that he did not guarantee payment but merely acted as an agent of the
take the benefits of the transactions as the person accordingly taxable." Federation which has a separate and distinct juridical personality. The Federation was
declared in default for failing to file an answer.
Thus, we repeat: to allow a taxpayer to deny tax liability on the ground that the sales were
made through another and distinct corporation when it is proved that the latter is virtually The trial court ruled in favor of the travel agency and held Kahn personally liable for the
owned by the former or that they are practically one and the same is to sanction a Federation’s obligation. It reasoned that Kahn failed to adduce proof of the corporate
circumvention of our tax laws. existence of the Federation, which was a mere sports association. Thus, a voluntary
unincorporated association, like the Federation has no power to enter into, or to ratify, a
contract. The contract entered into by its officers or agents on behalf of such association is
INTERNATIONAL EXPRESS TRAVEL & TOUR SERVICES, INC. vs. COURT OF APPEALS, HENRI
not binding on, or enforceable against it. The officers or agents are themselves personally
KAHN, PHILIPPINE FOOTBALL FEDERATION.
liable.
[G.R. No. 119002. October 19, 2000]
Only Henri Kahn elevated the above decision to the Court of Appeals. On 21 December
KAPUNAN, J.: 1994, the respondent court rendered a decision reversing the trial court. The Court of
Appeals recognized the juridical existence of the Federation and absolved Kahn from
personal liability. It rationalized that since petitioner failed to prove that Henri Kahn
guaranteed the obligation of the Federation, he should not be held liable for the same as
Facts: On June 30 1989, International Express Travel and Tour Services, Inc., wrote a said entity has a separate and distinct personality from its officers.
letter to the Philippine Football Federation’s (Federation) president Henri Kahn, offering its
services as a travel agency to the latter. The Federation secured the airline tickets for the
trips to the South East Asian Games in Kuala Lumpur as well as trips to the People's
Republic of China and Brisbane. The total cost of the tickets amounted to P449,654.83. For
Petitioner filed a motion for reconsideration and as an alternative prayer pleaded that the with the Federation in such a manner as to recognize and in effect admit its existence. The
Federation be held liable for the unpaid obligation. The same was denied by the appellate doctrine of corporation by estoppel is mistakenly applied by the respondent court to the
court on the grounds that the trial court dismissed the complaint against the federation, petitioner. The application of the doctrine applies to a third party only when he tries to
which was not appealed. Thus, the federation was not a party to this appeal. escape liability on a contract from which he has benefited on the irrelevant ground of
defective incorporation. In the case at bar, the petitioner is not trying to escape liability
Issue: Whether the Court of Appeal erred in finding that the Federation was a juridical from the contract but rather is the one claiming from the contract.
entity?

Held: Yes. The Court of Appeals cited Republic Act 3135, Revised Charter of the
Philippine Amateur Athletic Federation, and Presidential Decree No. 604 as the laws from WHEREFORE, the decision appealed from is REVERSED and SET ASIDE. The decision of the
which said Federation derives its existence. Above stated laws indicate that sports Regional Trial Court of Manila, Branch 35, in Civil Case No. 90-53595 is hereby REINSTATED.
associations, such as the Federation, may acquire a juridical personality. However,
national sports associations may be accorded corporate status, such does not
automatically take place by the mere passage of these laws.

Before a corporation may acquire juridical personality, the State must give its
consent either in the form of a special law or a general enabling act. We do not agree
with the appellate court that the Philippine Football Federation came into existence upon
the passage of these laws. Nowhere can it be found in R.A. 3135 or P.D. 604 any provision
creating the Philippine Football Federation.

Above stated laws require that before an entity may be considered as a national sports
association, such entity must be recognized by the accrediting organization, the Philippine
Amateur Athletic Federation under R.A. 3135, and the Department of Youth and Sports
Development under P.D. 604. This fact of recognition, however, Henri Kahn failed to
substantiate. He attempted to by attaching with motion for reconsideration before the
trial court a copy of the constitution and by-laws of the Federation. Unfortunately, that
does not prove that the Federation has been recognized and accredited. Accordingly, we
rule that the Philippine Football Federation is not a national sports association within the
purview of the aforementioned laws and does not have corporate existence of its own.
G.R. No. 116123 March 13, 1997

SERGIO F. NAGUIAT, doing business under the name and style SERGIO F. NAGUIAT ENT.,
Thus, Henry Kahn should be held liable for the unpaid obligations of the Federation. It is a
INC., & CLARK FIELD TAXI, INC., petitioners, vs. NATIONAL LABOR RELATIONS
settled principal in corporation law that any person acting or purporting to act on behalf
COMMISSION (THIRD DIVISION), NATIONAL ORGANIZATION OF WORKINGMEN and its
of a corporation which has no valid existence assumes such privileges and becomes
members, LEONARDO T. GALANG, et al., respondents.
personally liable for contract entered into or for other acts performed as such agent. As
president, Henri Kahn is presumed to have known about the corporate existence or non-
existence of the Federation. We cannot subscribe to the position taken by the appellate
court that even assuming that the Federation was defectively incorporated, the petitioner Nature: Special Civil Action for Certiorari under Rule 65 of the Rules of Court
cannot deny the corporate existence of the Federation because it had contracted and dealt
Facts: Petitioner Clark Field Taxi, Inc. (CFTI), the president of whom was Sergio Naguiat, were empolyees of Naguiat Enterprises. By virtue of the concessionaire's contract, CFTI
held a concessionaire's contract to operate a taxi service within Clark Air Base with Army purchased the fleet of vehicles from AAFES and became the owner thereof.
Air Force Exchange Services (AAFES). CFTI, like Sergio F. Naguiat Enterprises, was a family
corporation. For this purpose, petitioners hired private respondents as taxi drivers, Private respondents failed to substantiate their claim that Naguiat Enterprises managed,
working for at least 3 or 4 times in a week. supervised and controlled their employment. Further, in reading through the case, it seems
as if private respondents were merely confused as to the personalities of Sergio F. Naguiat
Due to the phase-out of US military bases in the Philippines, AAFES was dissolved. As a as an individual and as a separate corporate juridical entity. Closer scrutiny and analysis of
result, private respondents' services were terminated. Private respondents' drivers' union the records evince the truth that Sergio F. Naguiat, in supervising the taxi drivers and
and CFTI agreed to award separation benefits to their drivers in the amount of 500 pesos determining their employment terms, was carrying out his responsibilities as president of
for every year of service. Although majority of the members accepted their severance pay, CFTI. From the foregoing, the ineludible conclusion is that CFTI was the actual and direct
private respondents refused, subsequently disaffiliating from the drivers' union, joining the employer of individual respondents, and that Naguiat Enterprises was neither their indirect
National Organization of Workingmen (NOWM) and filing a complaint against petitioners employer nor labor-only contractor. It was not involved at all in the taxi business.
for payment of separation pay due to termination/phase out.
As to the liability of their officers, the Court ruled that in the the broader interest of justice,
Petitioner, by way of position paper, averred that the cessation of their business was due CFTI President Sergio Naguiat should be held liable. Following the ruling in A.C. Ransom
to great financial loss and lost business opportunities resulting from the phase-out. They Labor Union v. NLRC, the rule is that in the absence of definite proof as to which officer/s
further reiterated that CFTI had agreed with the drivers' union to award 500 pesos for should be held directly responsible for the payment of backwages, it should be presumed
every year of service as severance pay. that the responsible officer is the President of the corporation who can be deemed the
chief operation officer thereof.
The Labor Arbiter ruled in favor of private respondents, ordering petitioners to pay 1,200
pesos solidarily, to private respondents in lieu of separation pay for humanitarian Further, following the ruling in MAM Realty v. NLRC, a director or officer of a corporation
conditions. On appeal, the NLRC modified the decision of the Labor Arbiter and ordered maybe held liable solidarily if it is made by a specific provision of law. Section 100 of the
petitioners to pay separation pay in the amount agreed upon. Corporation Code specifically imposes personal liability upon the stockholder actively
managing or operating the business and affairs of the close corporation.
Issue: Whether or not the resolution of the NLRC was contrary to law

Wheter or not officers of corporations are ipso facto liable jointly and severally with the
companies they represent for the payment of separation pay. As to Antolin Naguiat, the Court ruled that he was not personally liable, for it had not been
shown that he had acted in the capacity of a general manager, as well as lack of evidence
Private respondents' contention: They are regular employees of Naguiat Enterprises, that his participation in the management and operation of the businesss was preferred.
despite their individual applications of employment were approved by CFTI, for the former
exercised control, management and supervision over their employment. Further, Naguiat
Enterprises, as indirect employer, is solidarily liable to pay them separation pay.
Ruling: (1) Petitioner Clark Field Taxi, Incorporated, and Sergio F. Naguiat, president and
Petitioners' contention: Sergio F. Naguiat Enterprises is a separate juridical entity that co-owner thereof, are ORDERED to pay, jointly and severally, the individual respondents
cannot be held solidarily liable. Further, Sergio and Antolin Naguiat, as President and Vice- their separation pay computed at US$120.00 for every year of service, or its peso
President of the Corporation, are merely officers of the same and cannot be held equivalent at the time of payment or satisfaction of the judgment;
personally liable.

Held: Naguiat Enterprises is not liable. Based on the factual findings of the parties, private
respondents were regular employees of CFTI who received wages on a boundary or (2) Petitioner Sergio F. Naguiat Enterprises, Incorporated, and Antolin T. Naguiat are
commission basis. There is, therefore, no substantial basis to hold Naguiat Enterprises as ABSOLVED from liability in the payment of separation pay to individual
an indirect employer. Sufficient evidence was shown that none of the private respondents respondents.
(3) appellee. Plaintiff-appellant was ordered to pay P5,103.35 to defendant-appellee within 90
days or the mortgaged property will be sold in a public auction.

CONTENTIONS

Petitioner-appellant: The appointment of the Bank of Taiwan as liquidator of the


HAW PIA, plaintiff-appellant, v. THE CHINA BANKING CORPORATION, defendant-appellee
defendant-appellee is valid under international laws. Since the Bank of Taiwan was the
GR No. L-554 || April 9, 1948 duly appointed liquidator of the defendant-appellee, tendering payment to it is equivalent
to tendering payment to the defendant-appellee itself.

This is an appeal of the decision of the trial court holding plaintiff-appellant Haw Pia liable
to pay the amount P5,103.35 to herein respondent China Banking Corporation. Plaintiff- Defendant-appellee: The liquidation of the China Banking Corporation and the subsequent
appellant prays that the defendant-appellee be compelled to execute a deed of appointment of the Bank of Taiwan as its liquidator are incorrect as the occupying
cancellation of the mortgaged property. Japanese forces had no authority to do so. Since there was no valid liquidation, any
payment made to the Bank of Taiwan cannot be deemed as payment made to the
defendant-appellee.

FACTS

ISSUES

Plaintiff-appellant Haw Pia incurred debts amounting to P5,103.35 from defendant-


appellee China Banking Corporation due to an overdraft in his current account during or
4.
before World War II when the Philippines was under occupation by the Japanese Imperial WON the liquidation of the China Banking Corporation and the subsequent appointment of
Army. During the occupation, the Japanese military authorities ordered the liquidation of the Bank of Taiwan as liquidator are valid;
China Banking Corporation as part of its efforts to stem Filipino resistance to the
occupation. They appointed and authorized the Bank of Taiwan, Ltd. as liquidator of5.the WON the payment made to the Bank of Taiwan as the liquidator of the China Banking
defendant-appellee. Since the defendant-appellee was in the process of liquidation, Corporation constitutes payment to China Banking Corporation itself; and
plaintiff-appellant paid his debt to the Bank of Taiwan. After the war, the defendant-
appellee instituted a court action seeking payment from the plaintiff-appellant of6.the How is this related to Nationality and Citizenship of Corporations (syllabus)?
P5,103.35 debt. The defendant-appellee took the position that the payment to the Bank of
Taiwan did not extinguish the obligation of the plaintiff-appellant to it. The trial court
rendered the assailed decision finding that the payment to the Bank of Taiwan did not
constitute payment to the defendant-appellee. According to the court, there was no RULING
evidence presented to show that the defendant-appellee authorized the Bank of Taiwan to
receive payment on its behalf. Furthermore, the Bank of Taiwan was an agent of the
Japanese Imperial Army and was not allowed to liquidate the business of the defendant-
The Supreme Court REVERSED the decision of the trial court. It ordered the defendant-
appellee to execute the deed of cancellation of mortgage of the mortgaged property and
to deliver to the plaintiff-appellant TCT No. 47634 with the annotation of mortgage therein The Supreme Court determined that China Banking Corporation qualified as an ENEMY
already cancelled. CORPORATION in the eyes of the Japanese military authorities. Under the Trading with the
Enemy Act, an enemy corporation is a corporation incorporated within such territory of
any nation with which the United States is at war. Applying the principle that what is
permitted to one belligerent is allowed to the other, the defendant-appellee was an enemy
1. to the Japanese. Not only was it controlled by Japan’s enemies, it was also incorporated
under the laws of a country with which Japan was at war.

The Supreme Court held that the Japanese military authorities had the power to order the Professional Services Inc v Agana
liquidation of the China Banking Corporation. The Hague Convention II which entered into
force on September 4, 1900 prohibits the confiscation of private properties of the
residents of the occupied territories by the occupying forces. Liquidation, however, cannot
be considered the same as confiscation. Sequestration contemplates a scenario where the These are three consolidated petitions for review on certiorari from the decision of the
occupying forces take possession of the properties of the citizens of the occupied Court of Appeals.
territories with the intention of conserving such properties. The sequestered properties
may be subject to further disposition by treaty between the belligerents at the end of war.
Liquidation serves a basic purpose during an occupation – to reduce the ability of the
Facts:
enemy to fight back. It is a part of economic warfare and has been practiced by major
powers. In fact, the United States has the Trading with the Enemy Act. Under the said Act, On April 4, 1984, Natividad Agana was rushed to the Medical City Hospital because of
the United States may order the liquidation, reorganization, and reopening of enemy banks difficulty of bowel movement and bloody anal discharge. After a series of medical
within occupied territories whenever appropriate. Since the United States has the Trading examinations, Dr. Miguel Ampil, diagnosed her to be suffering from "cancer of the
with the Enemy Act, it is presumed that Japan has an equivalent law under the principle sigmoid."
that “what is permitted to one belligerent is allowed to the other.” The liquidation of the
China Banking System and the appointment of a liquidator are therefore valid. On April 11, 1984, Dr. Ampil, assisted by the medical staff4 of the Medical City Hospital,
performed an anterior resection surgery on Natividad. He found that the malignancy in her
sigmoid area had spread on her left ovary, necessitating the removal of certain portions of
it. Thus, Dr. Ampil obtained the consent of Natividad’s husband, Enrique Agana, to permit
2.
Dr. Juan Fuentes, to perform hysterectomy on her. After Dr. Fuentes had completed the
hysterectomy, Dr. Ampil took over, completed the operation and closed the incision.
However, the operation appeared to be flawed. The records show that the nurse informed
The Supreme Court held that payment made to the Bank of Taiwan is equivalent to paying Dr. Ampil of 2 missing sponge but the doctor continued in closing the operation.
directly to the China Banking Corporation. Under Article 1162 (now 1240) of the then Civil
After a couple of days, Natividad complained of excruciating pain in her anal region. She
Code, payment shall be made to the person in whose favor the obligation was constituted,
consulted both Dr. Ampil and Dr. Fuentes about it. They told her that the pain was the
or his successors in interest, or any person authorized to receive it. The Bank of Taiwan, as
natural consequence of the surgery. Natividadthen went to the United States to seek
the liquidator of the defendant-appellee, is a person authorized to receive the payment.
further treatment. Natividad flew back to the Philippines, still suffering from pains. Two
weeks thereafter, her daughter found a piece of gauze protruding from her vagina. Upon
being informed about it, Dr. Ampil proceeded to her house where he managed to extract
3. by hand a piece of gauze measuring 1.5 inches in width. He then assured her that the pains
would soon vanish.
The pains intensified, prompting Natividad to seek treatment at the Polymedic General Ampil; and (3) it is not entitled to its counterclaim against the Aganas. PSI contends that Dr.
Hospital. While confined there, Dr. Ramon Gutierrez detected the presence of another Ampil is not its employee, but a mere consultant or independent contractor. As such, he
foreign object in her vagina -- a foul-smelling gauze measuring 1.5 inches in width which alone should answer for his negligence.
badly infected her vaginal vault. A recto-vaginal fistula had formed in her reproductive
organs which forced stool to excrete through the vagina. Thus, Natividad underwent The Aganas maintain that the Court of Appeals erred in finding that Dr. Fuentes is not
another surgery. guilty of negligence or medical malpractice, invoking the doctrine of res ipsa loquitur. They
contend that the pieces of gauze are prima facie proofs that the operating surgeons have
On November 12, 1984, Natividad and her husband filed with the RTC, Branch 96, Quezon been negligent.
City a complaint for damages against the Professional Services, Inc. (PSI), owner of the
Medical City Hospital, Dr. Ampil, and Dr. Fuentes. They alleged that the latter are liable for Dr. Ampil asserts that the Court of Appeals erred in finding him liable for negligence and
negligence for leaving two pieces of gauze inside Natividad’s body and malpractice for malpractice sans evidence that he left the two pieces of gauze in Natividad’s vagina. He
concealing their acts of negligence. pointed to other probable causes, such as: (1) it was Dr. Fuentes who used gauzes in
performing the hysterectomy; (2) the attending nurses’ failure to properly count the
On February 16, 1986, pending the outcome of the above cases, Natividad died and was gauzes used during surgery; and (3) the medical intervention of the American doctors who
duly substituted by her above-named children (the Aganas). examined Natividad in the United States of America.

On March 17, 1993, the RTC rendered its Decision in favor of the Aganas, finding PSI, Dr.
Ampil and Dr. Fuentes liable for negligence and malpractice.
Ruling:
Aggrieved, PSI, Dr. Fuentes and Dr. Ampil interposed an appeal to the Court of Appeals.
As to Dr. Ampil
The Court of Appeals rendered its decision stating that the case against Dr. Juan Fuentes is
dismissed and that PSI is solidarily liable with Dr. Ampil. Records show that he did not present any evidence to prove that the American doctors
were the ones who put or left the gauzes in Natividad’s body. Neither did he submit
evidence to rebut the correctness of the record of operation, particularly the number of
gauzes used. As to the alleged negligence of Dr. Fuentes, we are mindful that Dr. Ampil
Issues: examined Dr. Fuentes’ work and found it in order.

WON the Court of Appeals erred in holding Dr. Ampil liable for negligence and malpractice Dr. Ampil did not inform Natividad about the missing two pieces of gauze. Worse, he even
misled her that the pain she was experiencing was the ordinary consequence of her
WON the Court of Appeals erred in absolving Dr. Fuentes of any liability (this is not related operation. To our mind, what was initially an act of negligence by Dr. Ampil has ripened
anymore to the topic so I did not include it in the ruling. But if you want to know how the into a deliberate wrongful act of deceiving his patient.
SC ruled in this regard, they said that Dr. Fuentes is not liable because the operating doctor
was Dr Ampil who checked his (Dr Fuentes’) work after he was done and continued the As to PSI
operation himselfa(Dr Ampil).
There is employer-employee relationship between PSI and Dr. Ampil. Private hospitals,
WON PSI may be held solidarily liable for the negligence of Dr. Ampil. hire, fire and exercise real control over their attending and visiting ‘consultant’ staff.

PSI’s liability may also be gleaned upon the agency principle of apparent authority or
agency by estoppel and the doctrine of corporate negligence. Apparent authority, or what
Claims of the parties: is sometimes referred to as the "holding out" theory, or doctrine of ostensible agency or
agency by estoppel, has its origin from the law of agency. It imposes liability, not as the
PSI alleged in its petition that the Court of Appeals erred in holding that: (1) it is estopped
result of the reality of a contractual relationship, but rather because of the actions of a
from raising the defense that Dr. Ampil is not its employee; (2) it is solidarily liable with Dr.
principal or an employer in somehow misleading the public into believing that the public. Accordingly, it has the duty to exercise reasonable care to protect from harm all
relationship or the authority exists. The concept is essentially one of estoppel and has been patients admitted into its facility for medical treatment. Unfortunately, PSI failed to
explained in this manner: perform such duty.

"The principal is bound by the acts of his agent with the apparent authority which he It is worthy to note that Dr. Ampil and Dr. Fuentes operated on Natividad with the
knowingly permits the agent to assume, or which he holds the agent out to the public as assistance of the Medical City Hospital’s staff, composed of resident doctors, nurses, and
possessing. The question in every case is whether the principal has by his voluntary act interns. As such, it is reasonable to conclude that PSI, as the operator of the hospital, has
placed the agent in such a situation that a person of ordinary prudence, conversant with actual or constructive knowledge of the procedures carried out, particularly the report of
business usages and the nature of the particular business, is justified in presuming that the attending nurses that the two pieces of gauze were missing. The failure of PSI, despite
such agent has authority to perform the particular act in question. the attending nurses’ report, to investigate and inform Natividad regarding the missing
gauzes amounts to callous negligence. Not only did PSI breach its duties to oversee or
In this case, PSI publicly displays in the lobby of the Medical City Hospital the names and supervise all persons who practice medicine within its walls, it also failed to take an active
specializations of the physicians associated or accredited by it, including those of Dr. Ampil step in fixing the negligence committed. This renders PSI, not only vicariously liable for the
and Dr. Fuentes. We concur with the Court of Appeals’ conclusion that it "is now estopped negligence of Dr. Ampil under Article 2180 of the Civil Code, but also directly liable for its
from passing all the blame to the physicians whose names it proudly paraded in the public own negligence under Article 2176.
directory leading the public to believe that it vouched for their skill and competence."
Indeed, PSI’s act is tantamount to holding out to the public that Medical City Hospital, Anent the corollary issue of whether PSI is solidarily liable with Dr. Ampil for damages, let it
through its accredited physicians, offers quality health care services. By accrediting Dr. be emphasized that PSI, apart from a general denial of its responsibility, failed to adduce
Ampil and Dr. Fuentes and publicly advertising their qualifications, the hospital created the evidence showing that it exercised the diligence of a good father of a family in the
impression that they were its agents, authorized to perform medical or surgical services for accreditation and supervision of the latter. In neglecting to offer such proof, PSI failed to
its patients. As expected, these patients, Natividad being one of them, accepted the discharge its burden under the last paragraph of Article 2180 cited earlier, and, therefore,
services on the reasonable belief that such were being rendered by the hospital or its must be adjudged solidarily liable with Dr. Ampil. Moreover, as we have discussed, PSI is
employees, agents, or servants. also directly liable to the Aganas.

Corporate entities, like PSI, are capable of acting only through other individuals, such as WHEREFORE, we DENY all the petitions and AFFIRM the challenged Decision of the Court of
physicians. If these accredited physicians do their job well, the hospital succeeds in its Appeals.
mission of offering quality medical services and thus profits financially. Logically, where
negligence mars the quality of its services, the hospital should not be allowed to escape
liability for the acts of its ostensible agents.

We now proceed to the doctrine of corporate negligence or corporate responsibility.

One allegation in the complaint is that PSI as owner, operator and manager of Medical City
Hospital, "did not perform the necessary supervision nor exercise diligent efforts in the
supervision of Drs. Ampil and Fuentes and its nursing staff, resident doctors, and medical
interns who assisted Drs. Ampil and Fuentes in the performance of their duties as
surgeons." Premised on the doctrine of corporate negligence, the trial court held that PSI is BACHE & CO. (PHIL.), INC. and FREDERICK E. SEGGERMAN, petitioners, vs. HON. JUDGE
directly liable for such breach of duty. VIVENCIO M. RUIZ, MISAEL P. VERA, in his capacity as Commissioner of Internal Revenue,
ARTURO LOGRONIO, RODOLFO DE LEON, GAVINO VELASQUEZ, MIMIR DELLOSA,
We agree with the trial court. NICANOR ALCORDO, JO

In the present case, it was duly established that PSI operates the Medical City Hospital for (G.R. No. L-32409, 27 February 1971)
the purpose and under the concept of providing comprehensive medical services to the
This is an original action of certiorari, prohibition and mandamus, with prayer for a writ of RESPONDENT: Respondents, thru the Solicitor General, filed an answer to the petition.
preliminary mandatory and prohibitory injunction. They contended that a corporation is not entitled to protection against unreasonable
search and seizures

FACTS: On 24 February 1970, Misael P. Vera, Commissioner of Internal Revenue, wrote a


letter addressed to Judge Vivencio M. Ruiz requesting the issuance of a search warrant ISSUE: Whether or not the corporation has the right to object against unreasonable
against Bache & Co. (Phil.), Inc. and Frederick E. Seggerman for violation of Section 46(a) of searches and seizures?
the National Internal Revenue Code (NIRC), in relation to all other pertinent provisions
thereof, particularly Sections 53, 72, 73, 208 and 209, and authorizing Revenue Examiner
Rodolfo de Leon to make and file the application for search warrant which was attached to
the letter. In the afternoon of the following day, De Leon and his witness, Arturo Logronio, RULING: It is well settled that the legality of a seizure can be contested only by the party
went to the Court of First Instance (CFI) of Rizal. They brought with them the following whose rights have been impaired thereby, and that the objection to an unlawful search
papers: Vera’s letter-request; an application for search warrant already filled up but still and seizure is purely personal and cannot be availed of by third parties. Consequently,
unsigned by De Leon; an affidavit of Logronio subscribed before De Leon; a deposition in petitioners herein may not validly object to the use in evidence against them of the
printed form of Logronio already accomplished and signed by him but not yet subscribed; documents, papers and things seized from the offices and premises of the corporations
and a search warrant already accomplished but still unsigned by Judge. At that time the adverted to above, since the right to object to the admission of said papers in evidence
Judge was hearing a certain case; so, by means of a note, he instructed his Deputy Clerk of belongs exclusively to the corporations, to whom the seized effects belong, and may not
Court to take the depositions of De Leon and Logronio. After the session had adjourned, be invoked by the corporate officers in proceedings against them in their individual
the Judge was informed that the depositions had already been taken. The stenographer, capacity. In Stonehill, et al. vs. Diokno, et al., the Supreme Court impliedly recognized the
upon request of the Judge, read to him her stenographic notes; and thereafter, the Judge right of a corporation to object against unreasonable searches and seizures; holding that
asked Logronio to take the oath and warned him that if his deposition was found to be the corporations have their respective personalities, separate and distinct from the
false and without legal basis, he could be charged for perjury. The Judge signed de Leon’s personality of the corporate officers, regardless of the amount of shares of stock or the
application for search warrant and Logronio’s deposition. Search Warrant 2-M-70 was then interest of each of them in said corporations, whatever, the offices they hold therein may
signed by Judge and accordingly issued. Three days later (a Saturday), the BIR agents be; and that the corporate officers therefore may not validly object to the use in evidence
served the search warrant to the corporation and Seggerman at the offices of the against them of the documents, papers and things seized from the offices and premises of
corporation on Ayala Avenue, Makati, Rizal. the corporations, since the right to object to the admission of said papers in evidence
belongs exclusively to the corporations, to whom the seized effects belong, and may not
be invoked by the corporate officers in proceedings against them in their individual
capacity. The distinction between the Stonehill case and the present case is that: in the
PETITIONER: On March 3, 1970, petitioners, Bache & Co. (Phil.), Inc., a corporation duly former case, only the officers of the various corporations in whose offices documents,
organized and existing under the laws of the Philippines, and its President, Frederick E. papers and effects were searched and seized were the petitioners; while in the latter, the
Seggerman, filed a petition with the Court of First Instance of Rizal praying that the search corporation to whom the seized documents belong, and whose rights have thereby been
warrant be quashed, dissolved or recalled, that preliminary prohibitory and mandatory impaired, is itself a petitioner. On that score, petitioner corporation here stands on a
writs of injunction be issued, that the search warrant be declared null and void, and that different footing from the corporations in Stonehill.
the respondents be ordered to pay petitioners, jointly and severally, damages and
attorney’s fees. Petitioners provided for the following reasons: first, Respondent Judge Hence, the petition is granted and the search warrant is declared null and void.
failed to personally examine the complainant and his witness; second, the search warrant
was issued for more than one specific offense; and third, the search warrant does not
particularly describe the things to be seized.
P2.7M. This was also paid on the due date. Again, the bank extended another loan to the
corporation in the amount of P1M, covered by four promissory notes. However, the
corporation was unable to pay this at maturity. Thereupon, the bank applied for an extra-
judicial foreclosure of mortgage. For its part, the corporation filed an action for injunction
with prayer for damages. The lower court ultimately dismissed the case and ordered the
extra-judicial foreclosure of mortgage. Hence, this appeal.

PETITIONER

Petitioner corporation's contends that the case should be remanded to the trial court for a
[G.R. No. 103576. August 22, 1996]
specific finding on the amount of damages it has sustained "as a result of the unlawful
action taken by respondent bank against it.” This prayer is not reflected in its complaint
which has merely asked for the amount of P3,000,000.00 by way of moral damages.

ACME SHOE, RUBBER & PLASTIC CORPORATION and CHUA PAC, petitioners, PRIVATE RESPONDENT

vs. Private Respondent filed a motion to dismiss the petition.

HON. COURT OF APPEALS, PRODUCERS BANK OF THE PHILIPPINES and

REGIONAL SHERIFF OF CALOOCAN CITY, respondents. ISSUE: 1) Whether or not extra-judicial foreclosure of the chattel mortgage is proper?

2) If not proper, whether or not the corporation is entitled to damages as a result of the
extra-judicial foreclosure?

This is a petition for certiorari to set aside the decision of the appellate court affirming the
lower court’s decision which dismissed the complaint for damages filed by the petitioner RULING: 1) No. A chattel mortgage can only cover obligations existing at the time the
corporation and ordered the extra-judicial foreclosure of the chattel mortgage. mortgage is constituted. Although a promise expressed in a chattel mortgage to include
debts that are yet to be contracted can be a binding commitment that can be compelled
upon, the security itself, however, does not come into existence or arise until after a
chattel mortgage agreement covering the newly contracted debt is executed either by
FACTS: Chua Pac, president and general manager of Acme Shoe, Rubber and Plastic concluding a fresh chattel mortgage or by amending the old contract conformably with the
Corporation, executed a chattel mortgage in favor of Producers Bank of the Philippines, as form prescribed by the Chattel Mortgage Law. Refusal on the part of the borrower to
a security for a corporate loan in the amount of P3M. The chattel mortgage contained a execute the agreement so as to cover the after-incurred obligation can constitute an act of
clause that provided for the mortgage to stand as security for all other obligations default on the part of the borrower of the financing agreement whereon the promise is
contracted before, during and after the constitution of the mortgage. The P3M was paid.
Subsequently, the corporation obtained additional financial accommodations totaling
written but, of course, the remedy of foreclosure can only cover the debts extant at the WHEREFORE, the questioned decisions of the appellate court and the lower court are set
time of constitution and during the life of the chattel mortgage sought to be foreclosed. aside without prejudice to the appropriate legal recourse by private respondent as may
still be warranted as an unsecured creditor. No costs.

In Belgian Catholic Missionaries, Inc., vs. Magallanes Press, Inc., et al., the Court said -

FACTS: Petitioner Filipinas Broadcasting Network, Inc (FBNI) assails the Resolution of the
CA which modified the December 14, 1992 decision of the RTC of Legazpi City (as to the
"x x x A mortgage that contains a stipulation in regard to future advances in the credit will amount of moral damages), and found petitioner and its broadcasters Hermogenes Alegre
take effect only from the date the same are made and not from the date of the mortgage.” and Carmelo Rima liable for libel. The lower court ordered FBNI, Alegre and Rima to
solidarily pay moral damages, attorney’s fees and the costs of the suit to Ago Medical and
Educational Center- Bicol Christian College of Medicine (AMEC).

The significance of the ruling to the instant problem would be that since the 1978 chattel
mortgage had ceased to exist coincidentally with the full payment of the P3,000,000.00
loan, there no longer was any chattel mortgage that could cover the new loans that were The complaint alleged that Alegre and Rima had made “malicious imputations and as such
concluded thereafter. destroyed plaintiffs’ (AMEC and Angelita Ago, Dean of the College of Medicine) reputation”
by citing the alleged complaints of students, parents and teachers. The complaint cited
that defendants had made the ff libellous imputations with no factual basis: (1) That
AMEC-BCCM requires its students to repeat subjects which they have passed already the
Therefore, the extra-judicial foreclosure of the chattel mortgage was not proper.
moment they fail one subject, despite the absence of any such regulation by the DECS; (2)
That students were required to take and pay for the subject even if there is no instructor,
which demonstrates the greed of AMEC’s administration and; (3) That AMEC is a dumping
2) No. Even though the Court rendered that such extra-judicial foreclosure of chattel ground of moral and physical misfits because it continued to accept “rejects” in order to
mortgage was not proper, the petitioner corporation is not entitled to damages. minimize salary expenses. FBNI was impleaded as a defendant for “failing to exercise due
diligence in the selection and supervision of its employees (Alegre and Rima). A Motion to
Dismiss was filed in behalf of FBNI which the RTC denied. The lower court held that the
broadcasts were liable per se and were not the result of straight reporting because it had
In LBC Express, Inc. vs. Court of Appeals, we have said: no factual basis because the broadcasters failed to verify their reports. FBNI failed to
exercise the due diligence as required by law. Hence, the judgment requiring FBNI, Alegre
and Rima to pay moral damages (Php 300,000), plus reimbursement of attorney’s fees (Php
30,000) and the costs of the suit. CA lowered the amount of moral damages to Php
"Moral damages are granted in recompense for physical suffering, mental anguish, fright,
150,000.
serious anxiety, besmirched reputation, wounded feelings, moral shock, social humiliation,
and similar injury. A corporation, being an artificial person and having existence only in
legal contemplation, has no feelings, no emotions, no senses; therefore, it cannot
experience physical suffering and mental anguish. Mental suffering can be experienced ISSUE: Whether or not AMEC-BCCM, a corporation, is entitled to the award of moral
only by one having a nervous system and it flows from real ills, sorrows, and griefs of life - damages.
all of which cannot be suffered by respondent bank as an artificial person."
RULING: AMEC-BCCM is entitled to the award of moral damages. Generally, a juridical
person is not entitled to moral damages because, unlike a natural person, it cannot
experience physical suffering or such sentiments as wounded feelings, serious anxiety, Petitioners Francisco Tatad, John Osmena and Rodolfo Biazon are members of the
mental anguish or moral shock. In Mambulao Lumber Co. v. PNB, et al, the award of moral Philippine Senate and are suing in their capacities as Senators and as taxpayers.
damages may be justified. However, the Court’s statement in the said case, that “a Respondent Jesus Garcia was then Secretary of the DOTC, while private respondent EDSA
corporation may have a good reputation which, if besmirched, may also be a ground for LRT CORPORATION, Ltd. is a private corporation organized under the laws of Hongkong.
the award of moral damages”, is an obiter dictum.

In 1989, DOTC planned to construct a light railway transit line along EDSA, which shall
AMEC’s claim for moral damages was grounded under item 7 of Article 2219 of the Civil traverse the cities of Pasay, Quezon, Mandaluyong and Makati. The objective is to provide
Code which authorizes the same in cases of libel, slander, or any other form of defamation. a mass transit system along EDSA and to alleviate the congestion in the metropolis.
The provision does not qualify whether the plaintiff seeking such award is a natural or
juridical person. Therefore, a juridical person such as a corporation can validly complain of
libel or any other form of defamation and claim for moral damages as a result thereof.
On March 15, 1990, then DOTC Secretary Oscar Orbos, acting upon a proposal to construct
Moreover, evidence of an honest mistake or the want of character or reputation of the
the EDSA LRT III on a Build-Operate-Transfer (BOT) basis, had invited Elijahu Levin from the
party libelled serves only to mitigate the amount of damages. Since the broadcasts are
Eli Levin Enterprises, Inc to send a technical team to discuss the project with the DOTC.
libellous per se, AMEC is entitled to moral damages. The amount is reduced to Php 150,000
because AMEC has not suffered any substantial or material damage to its reputation.

G.R. No. 114222 April 6, 1995 On July 9, 1990, RA No. 6957 referred to as the Build-Operate-Transfer (BOT) was signed by
then President Corazon Aquino. The said Act provides for two schemes for the financing,
construction and operation of government projects through private initiative and
investment: BOT or Build-Transfer (BT).
FRANCISCO S. TATAD, JOHN H. OSMENA and RODOLFO G. BIAZON, petitioners,

vs.
In accordance with the provisions of RA 6957 and to set the EDSA LRT III project underway,
HON. JESUS B. GARCIA, JR., in his capacity as the Secretary of the Department of
the Prequalification Bids and Awards Committee and the Technical Committee were
Transportation and Communications, and EDSA LRT CORPORATION, LTD., respondents.
formed.
QUIASON, J.:

The prequalification criteria totalling 100% are as follows: a.) Legal aspects – 10%; b.)
Management/Organizational capability – 30%; c.) Financial capability- 30%; and d.)
DOCTRINE:
Technical capability – 30%.
This is a petition under Rule 65 of the Revised Rules of Court to prohibit respondents from
further implementing the “Revised and Restated Agreement to Build, Lease and Transfer a
Light Rail Transit System for EDSA and the Supplemental Agreement to the same project.
Of the 5 applicants, only the EDSA LRT Consortium, composed of CKD Tatra of the Czech
and Slovak Federal Republics, TCGI Engineering All Asia Capital and Leasing Corporation,
The Salim Group of Jakarta, E. L. Enterprises, Inc., A.M. Oreta & Co. Capitol Industrial
Facts: Construction Group, Inc, and F. F. Cruz & co., Inc, “met the requirements of garnering at
least 21 points per criteria, except for Legal aspects, and obtaining an over-all passing mark According to the agreements, the EDSA LRT III will use light rail vehicles from the Czech and
of at least 82 points.” The Legal aspects referred to provided that the BOT/BT contractor- Slovak Federal Republics and will have a maximum carrying capacity of 450,000 passengers
applicant meet the requirements specified in the Constitution and other pertinent laws. a day. The system will have its own power facility. It will also have 13 passenger stations
and one depot in 16-hectare government property at North Avenue.

Subsequently, Sec. Orbos was appointed Executive Secretary to the President of the
Philippines and was replaced by Nicomedes Prado. The latter recommended the award of Private respondents shall undertake and finance the entire project required for a complete
the EDSA LRT III project to the sole complying bidder, the EDSA LRT Consortium, and operational light rail transit system. Target completion date is approximately 3 years from
requested for authority to negotiate with the said firm for the contract pursuant to the the implementation date of the contract. Upon full and partial completion and viability
BOT Law. Authority was granted to proceed with the negotiations. The EDSA LRT thereof, private respondent shall deliver the use and possession of the completed portion
Consortium submitted its proposal to DOTC. to DOTC which shall operate the same. DOTC shall pay private respondent rentals on aj
monthly basis through an Irrevocable Letter of Credit. The rentals shall be determined by
an independent and internationally accredited inspection firm to be appointed by the
parties.
Finding the proposal to be in compliance with the bid requirements, DOTC and EDSA LRT
Corporation, Ltd., in substitution of the EDSA LRT Consortium, entered into an “An
Agreement to Build, Lease and Transfer a Light Rail Transit System for EDSA” under the
terms of the BOT Law. As agreed upon, private respondent’s capital shall be recovered from the rentals to be paid
by the DOTC which, in turn, shall come from the earnings of the EDSA LRT III. After 25 years
and DOTC shall have completed payment of the rentals, ownership of the project shall be
transferred to the latter for a consideration of only US $1.00.
Secretary Prado, thereafter, requested presidential approval of the contract.

In their petition, petitioners argued that the agreement of April 22, 1992, as amended by
Exec. Sec. Franklin Drilon, who replaced Sec. Orbos, informed Sec. Prado that the President the Supplemental Agreement of May 6, 1993, in so far as it grants EDSA LRT
could not grant the requested approval for failure to comply with the requirements of the COPORTATION, LTD., a foreign corporation, the ownership of EDSA LRT III, a public utility,
BOT Law. violates the constitution, and hence, is unconstitutional. They contend that the EDSA LRT III
is a public utility, and the ownership and operation thereof is limited by the Constitution to
Filipino citizens and domestic corporations, not foreign corporations like private
In view whereof, Sec. Drilon, the DOTC and private respondent re-negotiated the respondent.
agreement. On April 22, 1992, the parties entered into a “Revised and Restated Agreement
to Build, Lease and Transfer and Light Rail Transit System for EDSA. On May 6, 1992, DOTC,
represented by Sec. Jesus Garcia, Sec. Prado and private respondent entered into a Issue:
Supplemental Agreement to the April Revised Agreement so as to clarify their respective
rights and responsibilities.

Whether or not the EDSA LRT III (foreign corporations) is a common carrier and owns the
public utility violating Sec 11, Art XII 1987 Philippine constitution?
The two agreements were approved by President Fidel Ramos.
Held: Since DOTC shall operate the EDSA LRT III, it shall assume all the obligations and liabilities
of a common carrier. For this purpose, DOTC shall indemnify and hold harmless private
No. respondent from any losses, damages, injuries or death which may be claimed in the
operation or implementation of the system, except losses, damages, injury or death due to
What private respondent owns are the rail tracks, rolling stocks like the coaches, rail defects in the EDSA LRT III on account of the defective condition of equipment or facilities
stations, terminals and the power plant, not a public utility. While a franchise is needed to or the defective maintenance of such equipment facilities.
operate these facilities to serve the public, they do not by themselves constitute a public
utility. What constitutes a public utility is not their ownership but their use to serve the
public.
Wherefore, the petition is DISMISSED.

Section 11 of Article XII of the Constitution provides:

No franchise, certificate or any other form of authorization for the operation of a public
utility shall be granted except to citizens of the Philippines or to corporations or
associations organized under the laws of the Philippines at least sixty per centum of whose
capital is owned by such citizens, nor shall such franchise, certificate or authorization be
exclusive character or for a longer period than 50 years.

The right to operate a public utility may exist independently and separately from the
ownership of the facilities thereof. One can own said facilities without operating them as a
public utility, or conversely, one may operate a public utility without owning the facilities
used to serve the public. The devotion of property to serve the public may be done by the
owner or by the person in control thereof who may not necessarily be the owner thereof.

While private respondent is the owner of the facilities necessary to operate the EDSA LRT
III, it admits that it is not enfranchised to operate a public utility. In view of this incapacity,
private respondent and DOTC agreed that on completion date, private respondent will
immediately deliver possession of the LRT system by of lease for 25 years, during which
period DOTC shall operate the same as a common carrier and private respondent shall
provide technical maintenance and repair services to DOTC.

In sum, private respondent will not run the light rail vehicles and collect fees from the
riding public. It will have no dealings with the public and the public will have no right to
demand any services from it.
general ordinary powers, and where there is nothing in the articles of incorporation which
prohibit such a sale.

FACTS: This action is an appeal to the Supreme Court from the judgment of the Court of
Appeals rendering by default joint and several liability against all of the defendants for the
Mead v. McCullough & Magsaysay v. CA sum of $3, 450.61 gold.

By Cher Alcantara in CORPO (VDean) 3S AY11-12 ·

Plaintiff and defendant organized the "Philippine Engineering and Construction Company,"
the incorporators being the only stockholders and also the directors of said company, with
Edit Doc general ordinary powers. Each of the stockholders paid into the company $2,000 mexican
currency in cash, with the exception of Mead, who turned over to the company personal
1. G.R. No. 6217 December 26, 1911 property in lieu of cash. Shortly after the organization, the directors held a meeting and
elected the plaintiff as general manager. The plaintiff held this position with the company
for nine months, when he resigned to accept the position of engineer of the Canton and
Shanghai Railway Company. Under him, the company failed in their undertaking to raise a
CHARLES W. MEAD, plaintiff-appellant,
sunken Spanish fleet. It became a losing concern and a financial failure. Shortly after the
plaintiff left the Philippine Islands for China, the other directors, the defendants in this
vs.
case, held a meeting for the purpose of discussing the condition of the company at that
E. C. McCULLOUGH, ET AL., and THE PHILIPPINE ENGINEERING AND CONSTRUCTION time and determining what course to pursue. Thereafter, realizing that continuing the
COMPANY, defendant-appellants. operations of the company would mean more losses, the remaining directors unanimously
assigned all the rights and interests of the company to McCullough for value, who later also
assigned the same for value to other people who with McCullough subsequently formed
the Manila Salvage Association.

The plaintiff insists that he was received as general manager of the first company a salary,
profits made before the assignment and the value of the personal property which he have
Haussermann, Cohn & Fisher and A. D. Gibbs for plaintiff. left and sold to the defendants; while McCullough contends that the plaintiff was to
receive only his necessary expenses.
James J. Peterson and O'Brien & DeWitt for defendant McCullough.

ISSUE: Whether or not the remaining directors have the power to sell or transfer to one of
TRENT, J.: its members the assets of the corporation.

DOCTRINE: A majority of the stockholders or directors have the power to sell or transfer to RULING: YES.
one of its members the corporate property, where the stockholders or directors have
There were only five stockholders in this corporation at any time, four of whom were the
directors who made the sale, and the other the plaintiff, who was absent in China when
the said sale took place. The sale was, therefore, made by the unanimous consent of four- While a corporation remains solvent, we can see no reason why a director or officer, by the
fifths of all the stockholders. Under the articles of incorporation, the stockholders and authority of a majority of the stockholders or board of managers, may not deal with the
directors had general ordinary powers. There is nothing in said articles which expressly corporation, loan it money or buy property from it, in like manner as a stranger. So long as
prohibits the sale or transfer of the corporate property to one of the stockholders of said a purely private corporation remains solvent, its directors are agents or trustees for the
corporation. stockholders. They owe no duties or obligations to others. But the moment such a
corporation becomes insolvent, its directors are trustees of all the creditors, whether they
There is nothing in these articles which expressly or impliedly prohibits the sale of are members of the corporation or not, and must manage its property and assets with
corporate property to one of its members, nor a dissolution of a corporation in this strict regard to their interest; and if they are themselves creditors while the insolvent
manner. Neither is there anything in articles 151 to 174 of the Code of Commerce which corporation is under their management, they will not be permitted to secure to themselves
prohibits the dissolution of a corporation by such sale or transfer. by purchasing the corporate property or otherwise any personal advantage over the other
creditors. Nevertheless, a director or officer may in good faith and for an adequate
The articles of incorporation must include: consideration purchase from a majority of the directors or stockholders the property even
of an insolvent corporation, and a sale thus made to him is valid and binding upon the
The submission to the vote of the majority of the meeting of members, duly called and minority. The sale or transfer of the corporate property in the case at bar was made by
held, of such matters as may properly be brought before the same. (No. 10, art. 151, Code three directors who were at the same time a majority of stockholders. If a majority of the
of Commerce.) stockholders have a clear and a better right to sell the corporate property than a majority
of the directors, then it can be said that a majority of the stockholders made this sale or
Article XIII of the corporation's statutes expressly provides that "in all the meetings of the
transfer to the defendant McCullough.
stockholders, a majority vote of the stockholders present shall be necessary to determine
any question discussed."

The sale or transfer to one of its members was a matter which a majority of the The corporation had been going from bad to worse. The work of trying to raise the sunken
stockholders could very properly consider. But it is said that if the acts and resolutions of a Spanish fleet had been for several months abandoned. The corporation under the
majority of the stockholders in a corporation are binding in every case upon the minority, management of the plaintiff had entirely failed in this undertaking. It had broken its
the minority would be completely wiped out and their rights would be wholly at the mercy contract with the naval authorities and the $10,000 Mexican currency deposited had been
of the abuses of the majority. confiscated. It had no money. It was considerably in debt. It was a losing concern and a
financial failure. To continue its operation meant more losses. Success was impossible. The
Generally speaking, the voice of a majority of the stockholders is the law of the
corporation was civilly dead and had passed into the limbo of utter insolvency. The
corporation, but there are exceptions to this rule. There must necessarily be a limit upon
majority of the stockholders or directors sold the assets of this corporation, thereby
the power of the majority. Without such a limit the will of the majority would be absolute
relieving themselves and the plaintiff of all responsibility. This was only the wise and
and irresistible and might easily degenerate into an arbitrary tyranny. The reason for these
sensible thing for them to do. They acted in perfectly good faith and for the best interests
limitations is that in every contract of partnership (and a corporation can be something
of all the stockholders. "It would be a harsh rule that would permit one stockholder, or any
fundamental and unalterable which is beyond the power of the majority of the
minority of stockholders to hold a majority to their investment where a continuation of the
stockholders, and which constitutes the rule controlling their actions. this rule which must
business would be at a loss and where there was no prospect or hope that the enterprise
be observed is to be found in the essential compacts of such partnership, which gave
would be profitable."
served as a basis upon which the members have united, and without which it is not
probable that they would have entered not the corporation. Notwithstanding these
limitations upon the power of the majority of the stockholders, their (the majority's)
resolutions, when passed in good faith and for a just cause, deserve careful consideration
and are generally binding upon the minority.
Artemio Panganiban, Subic Land Corporation (SUBIC), Filipinas Manufacturer's Bank
(FILMANBANK) and the Register of Deeds of Zambales. In her complaint, she alleged that in
1958, she and her husband acquired, thru conjugal funds, a parcel of land with
improvements, known as "Pequena Island", covered by TCT No. 3258; that after the death
of her husband, she discovered [a] an annotation at the back of TCT No. 3258 that "the
land was acquired by her husband from his separate capital;" [b] the registration of a Deed
of Assignment dated June 25, 1976 purportedly executed by the late Senator in favor of
SUBIC, as a result of which TCT No. 3258 was cancelled and TCT No. 22431 issued in the
name of SUBIC; and [c] the registration of Deed of Mortgage dated April 28, 1977 in the
amount of P 2,700,000.00 executed by SUBIC in favor of FILMANBANK; that the foregoing
acts were void and done in an attempt to defraud the conjugal partnership considering
that the land is conjugal, her marital consent to the annotation on TCT No. 3258 was not
obtained, the change made by the Register of Deeds of the titleholders was effected
without the approval of the Commissioner of Land Registration and that the late Senator
did not execute the purported Deed of Assignment or his consent thereto, if obtained, was
G.R. No. 58168 December 19, 1989 secured by mistake, violence and intimidation. She further alleged that the assignment in
favor of SUBIC was without consideration and consequently null and void. She prayed that
CONCEPCION MAGSAYSAY-LABRADOR, SOLEDAD MAGSAYSAY-CABRERA, LUISA the Deed of Assignment and the Deed of Mortgage be annulled and that the Register of
MAGSAYSAY-CORPUZ, assisted be her husband, Dr. Jose Corpuz, FELICIDAD P. Deeds be ordered to cancel TCT No. 22431 and to issue a new title in her favor.
MAGSAYSAY, and MERCEDES MAGSAYSAY-DIAZ, petitioners, vs.THE COURT OF APPEALS
and ADELAIDA RODRIGUEZ-MAGSAYSAY, Special Administratrix of the Estate of the late On March 7, 1979, herein petitioners, sisters of the late senator, filed a motion for
intervention on the ground that on June 20, 1978, their brother conveyed to them one-half
Genaro F. Magsaysay respondents.
(1/2) of his shareholdings in SUBIC or a total of 416,566.6 shares and as assignees of
FERNAN, C.J.: around 41 % of the total outstanding shares of such stocks of SUBIC, they have a
substantial and legal interest in the subject matter of litigation and that they have a legal
interest in the success of the suit with respect to SUBIC but which the court denied the
motion for intervention ruling lack of legal interest for being alleged assignees of certain
DOCTRINE: Section 63 of the Corporation Code provides, thus: "No transfer, however, shall shares in SUBIC cannot legally entitle them to intervene because SUBIC has a personality
be valid, except as between the parties, until the transfer is recorded in the books of the separate and distinct from its stockholders.
corporation showing the names of the parties to the transaction, the date of the transfer,
the number of the certificate or certificates and the number of shares transferred." The Court of Appeals found no legal justification to disturb the findings of the lower court
and stated that petitioner’s claims can be ventilated in a separate proceeding. Petitioner’s
motion for reconsideration was denied. Hence this instant recourse. Petitioners argue that
41.66% of the entire outstanding capital stock of SUBIC entitles them to a significant vote
FACTS: In this petition for review on certiorari, petitioners seek to reverse and set aside CA on corporate affairs that they are affected by the action of the widow of their late brother
decision affirming that of the CFI of Zambales and Olongapo City denying petitioners' for it concerns the only tangible asset of the corporation and that it appears that they are
motion to intervene in an annulment suit filed by herein private respondent, and [2] its more vitally interested in the outcome of the case than SUBIC.
resolution denying their motion for reconsideration.

ISSUE: Whether or not petitioners have a legal interest in the subject matter in litigation
Adelaida Rodriguez-Magsaysay, widow and special administratix of the estate of the late to entitle them to intervene in the proceedings.
Senator Genaro Magsaysay, brought before the then CFI of Olongapo an action against
2. The petitioners cannot claim the right to intervene on the strength of the transfer of
shares allegedly executed by the late Senator. The corporation did not keep books and
HELD: No. records. 11 Perforce, no transfer was ever recorded, much less effected as to prejudice
third parties. The transfer must be registered in the books of the corporation to affect third
1. . In the case of Batama Farmers' Cooperative Marketing Association, Inc. v. Rosal, we persons. The law on corporations is explicit. Section 63 of the Corporation Code provides,
held: "As clearly stated in Section 2 of Rule 12 of the Rules of Court, to be permitted to thus: "No transfer, however, shall be valid, except as between the parties, until the
intervene in a pending action, the party must have a legal interest in the matter in transfer is recorded in the books of the corporation showing the names of the parties to
litigation, or in the success of either of the parties or an interest against both, or he must the transaction, the date of the transfer, the number of the certificate or certificates and
be so situated as to be adversely affected by a distribution or other disposition of the the number of shares transferred."
property in the custody of the court or an officer thereof ."
And even assuming arguendo that there was a valid transfer, petitioners are nonetheless
To allow intervention, [a] it must be shown that the movant has legal interest in the matter barred from intervening inasmuch as their rights can be ventilated and amply protected in
in litigation, or otherwise qualified; and [b] consideration must be given as to whether the another proceeding.
adjudication of the rights of the original parties may be delayed or prejudiced, or whether
the intervenor's rights may be protected in a separate proceeding or not. Both
On the topic of corporate criminal liability
requirements must concur as the first is not more important than the second.

The interest which entitles a person to intervene in a suit between other parties must be in
SIA v. PEOPLE OF THE PHILIPPINES
the matter in litigation and of such direct and immediate character that the intervenor will
either gain or lose by the direct legal operation and effect of the judgment. Otherwise, if GR No. L-30896
persons not parties of the action could be allowed to intervene, proceedings will become
April 28, 1983
unnecessarily complicated, expensive and interminable. And this is not the policy of the
law. De Castro, J.:

The words "an interest in the subject" mean a direct interest in the cause of action as
pleaded, and which would put the intervenor in a legal position to litigate a fact alleged in
the complaint, without the establishment of which plaintiff could not recover.
Facts:
Here, the interest, if it exists at all, of petitioners-movants is indirect, contingent, remote,
This case involves a petition for review of the decision of the Court of Appeals affirming the
conjectural, consequential and collateral. At the very least, their interest is purely inchoate,
CFI of Manila convicting the appellant of estafa. The facts reveal that in 1963,
or in sheer expectancy of a right in the management of the corporation and to share in the
the accused Jose Sia was the general manager of Metal
profits thereof and in the properties and assets thereof on dissolution, after payment of
Manufacturing Company of the Philippines engaged in the manufacturing of steel
the corporate debts and obligations.
office equipment. When the company was in need of raw materials to be
imported from abroad, Sia applied for a letter of credit to import steel
While a share of stock represents a proportionate or aliquot interest in the property of the
sheets from Tokyo, Japan, the application being
corporation, it does not vest the owner thereof with any legal right or title to any of the
directed to Continental Bank and was opened in the amount of $18,300. According
property, his interest in the corporate property being equitable or beneficial in nature.
to the Continental Bank, the delivery of the steel sheets was only permitted upon the
Shareholders are in no legal sense the owners of corporate property, which is owned by
execution of the trust receipt. While according to Sia, the steel sheets were
the corporation as a distinct legal person.
already delivered and were even
converted to equipment before the trust receipt was signed by him. However,
there is no question that when the bill of exchange became due, neither the accused nor
his company made payments, despite demands of the bank. On appeal, Sia contends that The complaint alleged that Alegre and Rima had made “malicious imputations and as such
he should not be held liable. destroyed plaintiffs’ (AMEC and Angelita Ago, Dean of the College of Medicine) reputation”
by citing the alleged complaints of students, parents and teachers. The complaint cited
that defendants had made the ff libellous imputations with no factual basis: (1) That
Issue: WON petitioner Sia may be liable for the crime charged, having acted only for and in AMEC-BCCM requires its students to repeat subjects which they have passed already the
behalf of his company. moment they fail one subject, despite the absence of any such regulation by the DECS; (2)
That students were required to take and pay for the subject even if there is no instructor,
which demonstrates the greed of AMEC’s administration and; (3) That AMEC is a dumping
Held: ground of moral and physical misfits because it continued to accept “rejects” in order to
minimize salary expenses. FBNI was impleaded as a defendant for “failing to exercise due
diligence in the selection and supervision of its employees (Alegre and Rima). A Motion to
NO. The Court disputed the reliance of the lower court and the CA on the general principle Dismiss was filed in behalf of FBNI which the RTC denied. The lower court held that the
that for a crime committed by a corporation, the responsible officers thereof would broadcasts were liable per se and were not the result of straight reporting because it had
personally bear the criminal liability, as enunciated in Tan Boon Kong. The latter provides no factual basis because the broadcasters failed to verify their reports. FBNI failed to
that: “[t]he corporation was directly required by law to do an act in a given manner and exercise the due diligence as required by law. Hence, the judgment requiring FBNI, Alegre
the same law makes the person who fails to perform the act in the prescribed manner and Rima to pay moral damages (Php 300,000), plus reimbursement of attorney’s fees (Php
expressly liable criminally. The performance of an act is an obligation directly imposed by 30,000) and the costs of the suit. CA lowered the amount of moral damages to Php
the law on the corporation. Since it is a responsible officer or officers of the corporations 150,000.
who actually perform the act for the corporation, they must of necessity be the ones to
assume the criminal liability; otherwise this liability as created by the law would be illusory,
and the deterrent effect of the law, negated. ISSUE: Whether or not AMEC-BCCM, a corporation, is entitled to the award of moral
damages.

The Court concluded that the cited case does not fall squarely with the circumst
ances surrounding Sia since the act alleged to be a crime is not in the performance of an RULING: AMEC-BCCM is entitled to the award of moral damages. Generally, a juridical
act directly ordained by law to be performed by the corporation. The act is imposed by the person is not entitled to moral damages because, unlike a natural person, it cannot
agreement of the parties in pursuit of the business. The intention of the parties is experience physical suffering or such sentiments as wounded feelings, serious anxiety,
therefore a factor determinant of whether a crime or a civil obligation alone is mental anguish or moral shock. In Mambulao Lumber Co. v. PNB, et al, the award of moral
committed. The absence of a provision of the law even in the RPC making Sia criminally damages may be justified. However, the Court’s statement in the said case, that “a
liable as the president of his company created a doubt that must be ruled in his favor corporation may have a good reputation which, if besmirched, may also be a ground for
according to the maxim, that all doubts must be resolved in favor of the accused. the award of moral damages”, is an obiter dictum.

Filipinas Broadcasting v. Ago Medical and Educational Center- Bicol Christian College of
Medicine G.R. No. 141994. January 17, 2005 AMEC’s claim for moral damages was grounded under item 7 of Article 2219 of the Civil
Code which authorizes the same in cases of libel, slander, or any other form of defamation.
FACTS: Petitioner Filipinas Broadcasting Network, Inc (FBNI) assails the Resolution of the
The provision does not qualify whether the plaintiff seeking such award is a natural or
CA which modified the December 14, 1992 decision of the RTC of Legazpi City (as to the
juridical person. Therefore, a juridical person such as a corporation can validly complain of
amount of moral damages), and found petitioner and its broadcasters Hermogenes Alegre
libel or any other form of defamation and claim for moral damages as a result thereof.
and Carmelo Rima liable for libel. The lower court ordered FBNI, Alegre and Rima to
Moreover, evidence of an honest mistake or the want of character or reputation of the
solidarily pay moral damages, attorney’s fees and the costs of the suit to Ago Medical and
party libelled serves only to mitigate the amount of damages. Since the broadcasts are
Educational Center- Bicol Christian College of Medicine (AMEC).
libellous per se, AMEC is entitled to moral damages. The amount is reduced to Php 150,000
because AMEC has not suffered any substantial or material damage to its reputation.
ATTY. MAURICIO ULEP vs THE LEGAL CLINIC, INC GUAM DIVORCE. DON PARKINSON

Bar Matter 553 / 223 SCRA 378 / June 17, 1993 an Attorney in Guam, is giving FREE BOOKS on Guam Divorce through The Legal Clinic
beginning Monday to Friday during office hours.
Justice Regalado
Guam divorce. Annulment of Marriage. Immigration Problems, Visa Ext. Quota/Non-quota
Res. & Special Retiree's Visa. Declaration of Absence. Remarriage to Filipina Fiancees.
DOCTRINE: Adoption. Investment in the Phil. US/Foreign Visa for Filipina Spouse/Children. Call Marivic.
THE 7F Victoria Bldg. 429 UN Ave., LEGAL Ermita, Manila nr. US Embassy CLINIC, INC. 1 Tel.
Practice of law means any activity, in or out of court, which requires the application of law,
521-7232; 521-7251; 522-2041; 521-0767
legal procedures, knowledge, training and experience. To engage in the practice of law is to
perform those acts which are characteristic of the profession. Generally, to practice law is
to give advice or render any kind of service that involves legal knowledge or skill.
2. Petitioner contends that the advertisements are champertous, unethical,
demeaning of the law profession, and destructive of the confidence of the community in
the integrity of the members of the bar and that, to which as a member of the legal
That fact that the corporation employs paralegals to carry out its services is not controlling.
profession, he is ashamed and offended.
What is important is that it is engaged in the practice of law by virtue of the nature of the
services it renders which thereby brings it within the ambit of the statutory prohibitions
against the advertisements which it has caused to be published and are now assailed in
3. Respondent claims it is not engaged in the practice of law, but in rendering “legal
this proceeding.
support services” through paralegals with the use of modern computers and electronic
machines. Further, they aver that assuming the services advertised are legal services, the
act of advertising them should be allowed by decided jurisprudence (John Bates & Van
[PETITION TO THE SC TO ISSUE CEASE & DESIST ORDER OF RESPONDENT’S
O’Steen vs State Bar of Arizona.
ADVERTISEMENTS]

ISSUE: Whether respondent corporation - The Legal Clinic - is engaged in the unauthorized
FACTS:
practice of law to merit the issuance of the cease & desist order.
1. Petitioner Mauricio Ulep prays for the SC to issue a cease & desist order to the
respondent, The Legal Clinic, Inc., to perpetually refrain them from advertising their
services, tagged as Annex A & B, to wit: RULING:

YES. Practice of law means any activity, in or out of court, which requires the application
of law, legal procedures, knowledge, training and experience. To engage in the practice of
Annex A
law is to perform those acts which are characteristic of the profession. Generally, to
SECRET MARRIAGE? P560.00 for a valid marriage. practice law is to give advice or render any kind of service that involves legal knowledge or
skill.
Info on DIVORCE. ABSENCE. ANNULMENT. VISA.

THE Please call: 521-0767 LEGAL 5217232, 5222041 CLINIC, INC.


The practice of law is not limited to the conduct of cases in court. It includes legal advice
8:30 am— 6:00 pm 7-Flr. Victoria Bldg., UN Ave., Mla.
and counsel, and the preparation of legal instruments and contract by which legal rights
are secured, although such matter may or may not be pending in a court…..Giving advice
for compensation regarding the legal status and rights of another and the conduct with
Annex B
respect thereto constitutes a practice of law.
That fact that the corporation employs paralegals to carry out its services is not controlling.
G.R. No. 15574 September 17, 1919
What is important is that it is engaged in the practice of law by virtue of the nature of the
services it renders which thereby brings it within the ambit of the statutory prohibitions SMITH, BELL & COMPANY (LTD.), petitioner,
against the advertisements which it has caused to be published and are now assailed in
vs.
this proceeding.
JOAQUIN NATIVIDAD, Collector of Customs of the port of Cebu, respondent.

Public policy requires that the practice of law be limited to those individuals found duly
qualified in education and character. The purpose is to protect the public, the court, the A writ of mandamus is prayed for by Smith, Bell & Co. (Ltd.), against Joaquin Natividad,
client and the bar from the incompetence or dishonesty of those unlicensed to practice law Collector of Customs of the port of Cebu, Philippine Islands, to compel him to issue a
and not subject to the disciplinary control of the court. certificate of Philippine registry to the petitioner for its motor vessel Bato. The Attorney-
General, acting as counsel for respondent, demurs to the petition on the general ground
that it does not state facts sufficient to constitute a cause of action.
It is apt to recall that only natural persons can engage in the practice of law, and such
limitation cannot be evaded by a corporation employing competent lawyers to practice for
it. Obviously, this is the scheme or device by which respondent "The Legal Clinic, Inc." Facts:
holds out itself to the public and solicits employment of its legal services. It is an odious
Smith, Bell & Co. is a corporation organized and existing under the laws of the Philippine
vehicle for deception, especially so when the public cannot ventilate any grievance
Islands; majority of the stockholders are British; owner of a motor vessel known as the
for malpractice against the business conduit. Precisely, the limitation of practice of law to
Bato—brought to Cebu for the purpose of transporting Smith, Bell & Co.’s merchandise
persons who have been duly admitted as members of the Bar (Sec. 1, Rule 138, Revised
between ports in the islands. Application for registration was made at Cebu at the
Rules of Court) is to subject the members to the discipline of the Supreme Court. Although
Collector of Customs---denied. Because they were not citizens of the US/Phils. Act 2671,
respondent uses its business name, the persons and the lawyers who act for it are subject
Sec. 1172. Certificate ofPhilippine Register. upon registration of a vessel of domestic
to court discipline. The practice of law is not a profession open to all who wish to engage in
ownership, and of more than 15 tons gross, a certificate of Philippine register shall be
it nor can it be assigned to another (See 5 Am. Jur. 270). It is a personal right limited to
issued for it. If the vessel is of domestic ownership and of 15 tons gross or less, the taking
persons who have qualified themselves under the law. It follows that not only respondent
of the certificate of Philippine register shall be optional with the owner. domestic
but also all the persons who are acting for respondent are the persons engaged in
ownership, as used in this section, means ownership vested in the (a) citizens or native
unethical law practice.
inhabitants of the Phil Islands; (b) citizens of the US residing in the Phil. Islands; (c) any
corporation or company composed wholly of citizen of Phils./US or both

plaintiff’s contention: Act No. 2671 deprives the corp. of its property without due process
of law

because by the passage of the law, the company was automatically deprived of every
beneficial

attribute of ownership of the Bato and that they are left with a naked title they could not
use.

Issue: WON Smith, Bell & Co. were denied of the due process of law by the Phil. Legislature
in its enactment of Act 2761.
conditions stipulated (Exhibit A). Pursuant thereto, on May 19,1958, NAMARCO delivered
to ASSOCIATED 7,732.71 bars of "Busilak" and 17,285.08 piculs of "Pasumil" domestic raw
Held:
sugar. As ASSOCIATED failed to deliver to NAMARCO the 22,516 bags of "Victoria" and/or
No. (judgment affirmed—plaintiff can’t be granted registry.) While Smith, Bell & Co. Ltd., a "National" refined sugar agreed upon, the latter, on January 12, 1959, demanded in writing
corporation having alien stockholders, is entitled to the protection afforded by the due- from the ASSOCIATED either (a) immediate delivery thereof before January 20, or (b)
process of law and equal protection of the laws clause of the Philippine Bill of Rights, payment of its equivalent cash value amounting to P372,639.80.
nevertheless, Act No. 2761 of the Philippine Legislature, in denying to corporations such as
Smith, Bell &. Co. Ltd., the right to register vessels in the Philippines coastwise trade, does
not belong to that vicious species of class legislation which must always be condemned, As ASSOCIATED refused to deliver the raw sugar or pay for the refund sugar delivered to it,
but does fall within authorized exceptions, notably, within the purview of the police power, inspite of repeated demands therefore, NAMARCO instituted the present action in the
and so does not offend against the constitutional provision. Literally and absolutely, lower court to recover the sum of P403,514.28 in payment of the raw sugar received by
steamship lines are the arteries of the commerce in the Phils. If one be severed, the defendants from it; P80,702.86 as liquidated damages; P10,000.00 as attorney’s fees,
lifeblood of the nation is lost. If these are protected, security of the country and general expenses of litigation and exemplary damages, with legal interest thereon from the filing of
welfare is sustained. the complaint until fully paid.

NATIONAL MARKETING CORPORATION V. ASSOCIATED FINANCE COMPANY, INC. and


FRANCISCO SYCIP
In their amended answer defendants, by way of affirmative defenses, alleged that the
No. L-20886 correct value of the sugar delivered by NAMARCO to them was P259,451.09 or P13.30 per
bag of 100 lbs. weight (quedan basis) and not P403,514.28 as claimed by NAMARCO. As
April 27, 1967
counterclaim they prayed for the award of P500,000.00 as moral damages, P100,000.00 as
exemplary damages and P10,000.00 as attorney’s fee.

DIZON, J.:

Appeal taken by the National Marketing Corporation from the decision of the Court of First ISSUE:The only issue to be resolved is whether, upon the facts found by the trial court,
Instance of Manila in Civil Case No. 45770 ordering the Associated Finance Company, Inc. Francisco Sycip may be held liable, jointly and severally with his co-defendant, for the sums
to pay the NAMARCO the sum of P403,514.28, with legal interest thereon from the date of of money adjudged in favor of NAMARCO.
filing of the action until fully paid, P80,702.26 as liquidated damages, P5,000.00 as
attorney’s fee, plus costs, but dismissing the complaint insofar as defendant Francisco
Sycip was concerned, as well as the latter’s counterclaim. The appeal is only from that RULING:The foregoing facts, fully established by the evidence, can lead to no other
portion of the decision dismissing the case as against Francisco Sycip. conclusion than that Sycip was guilty of fraud because through false representations he
succeeded in including NAMARCO to enter into the aforesaid exchange agreement, with
full knowledge, on his part, of the fact that ASSOCIATED whom he represented and over
FACTS: whose business and affairs he had absolute control, was in no position to comply with the
obligation it had assumed. Consequently, he cannot now seek refuge behind the general
principle that a corporation has a personality distinct and separate from that of its
ASSOCIATED, a domestic corporation, through its President, appellee Francisco Sycip, stockholders and that the latter are not personally liable for the corporate obligations. To
entered into an agreement to exchange sugar with NAMARCO, represented by its then the contrary, upon the proven facts, we feel perfectly justified in “piercing the veil of
General Manager, Benjamin Estrella, whereby the former would deliver to the latter corporate fiction” and in holding Sycip personally liable, jointly and severally with his co-
22,516 bags (each weighing 100 pounds) of "Victorias" and/or "National" refined sugar in defendant, for the sums of money adjudged in favor of appellant. It is settled law in this
exchange for 7,732.71 bags of "Busilak" and 17,285.08 piculs of "Pasumil" raw sugar and other jurisdictions that when the corporation is the mere alter ego of a person, the
belonging to NAMARCO, both agreeing to pay liquidated damages equivalent to 20% of the corporate fiction may be disregarded; the same being true when the corporation is
contractual value of the sugar should either party fail to comply with the terms and
controlled, and its affairs are so conducted as to make it merely an instrumentality, agency the said certificates. Both the applications of the Corporation and Pantranco were set for
or conduit of another. joint hearing.

VILLA REY TRANSIT, INC., plaintiff-appellant, vs. EUSEBIO E. FERRER, PANGASINAN The PSC issued an order disposing that during the pendency of the cases Pantranco
TRANSPORTATION CO., INC. and PUBLIC SERVICE COMMISSION, defendants. EUSEBIO E. shall be the one to operate provisionally the service The Corporation elevated the matter
FERRER and PANGASINAN TRANSPORTATION CO., INC., defendants-appellants. to the Supreme Court. On November 4, 1959, the Corporation filed in the Court of First
Instance of Manila, a complaint for the annulment of the sheriff's sale to Ferrer, the latter’s
sale to Pantranco and PSC decision regarding the issue.
PANGASINAN TRANSPORTATION CO., INC., third-party plaintiff-appellant, vs. JOSE M.
Ferrer and Pantranco averred that the Corporation had no valid title to the certificates in
VILLARAMA, third-party defendant-appellee.
question because the contract pursuant to which it acquired them from Fernando was
subject to a suspensive condition, the approval of the PSC, has not yet been fulfilled. Thus
they believed that their purchase through the sheriff afforded them a better right.
G.R. No. L-23893 October 29, 1968
Pantranco, filed a third-party complaint against Jose M. Villarama, alleging that Villarama
ANGELES, J.:
and the Corporation, are one and the same; that Villarama and/or the Corporation was
Facts: Jose M. Villarama was an operator of a bus transportation, Villa Rey Transit, with disqualified from operating the two certificates in question by virtue of the agreement
certificates of public convenience granted by the Public Service Commission (PSC) in Cases between Villarama and Pantranco, stipulating that Villarama "shall not for a period of 10
Nos. 44213 and 104651, authorizing him to operate 32 units on various routes or lines years from the date of this sale, apply for any TPU service identical or competing with the
from Pangasinan to Manila, and vice-versa. On January 8, 1959, he sold the buyer."
aforementioned two certificates of public convenience to the Pangasinan Transportation
The CFI ruled in favor of the Corporation and Villarama. It held that the sheriff’s sale
Company, Inc. (Pantranco), for P350,000.00 with the condition that Villarama "shall not for
was void, that the Corporation was the lawful owner of the certificates and ordering Ferrer
a period of 10 years from the date of this sale, apply for any TPU service identical or
and Pantranco to pay attorney’s fees. It also held that Villarama and the corporation were
competing with the buyer."
separate and distinct entities.
Barely three months thereafter, on March 6, 1959: a corporation, Villa Rey Transit,
Issues: (1) Whether the agreement that Villarama "SHALL NOT FOR A PERIOD OF 10 YEARS
Inc. (Corporation) was organized; Natividad R. Villarama (wife of Jose M. Villarama) was
FROM THE DATE OF THIS SALE, APPLY FOR ANY TPU SERVICE IDENTICAL OR COMPETING
one of the incorporators, and the brother and sister-in-law of Jose M. Villarama subscribed
WITH THE BUYER," apply to new lines only or does it include existing lines?;
to the stock. Natividad also became the treasurer of the corporation. On March 10, 1959
the corporation was registered with the SEC. On April 7, 1959, the Corporation bought five (2) Assuming that said stipulation covers all kinds of lines, is such stipulation valid and
certificates of public convenience, forty-nine buses, tools and equipment from one enforceable?;
Valentin Fernando. They immediately applied with the PSC for its approval, praying for
(3) In the affirmative, that said stipulation is valid, did it bind the Corporation?
provisional authority to operate the service. On May 19, 1959, the PSC granted the
provisional permit prayed for. Before the PSC could take final action on said application for
approval of sale, however, the Sheriff of Manila, on July 7, 1959, levied on two of the five
Held: Although Villarama was not a stockholder or an incorporator, his wife was an
certificates of public convenience involved therein, pursuant to a writ of execution issued
incorporator and also the treasurer of the Corporation. The evidence proved that
by the Court of First Instance of Pangasinan, in favor of Eusebio Ferrer, judgment creditor,
Villarama had actual control of the funds of the Corporation and appeared as the actual
against Valentin Fernando. On July 16, 1959, a public sale was conducted, with Ferrer as
owner and treasurer. In fact the funds of the Corporation were deposited in his personal
the highest bidder.
account. The initial cash capitalization of P105,000 was mostly financed by Villaram
through an P85,000 personal check he issued himself. The trucks of the Corporation were
also purchased with his personal checks. Gasoline purchases were made in his name. His
Ferrer sold the two certificates of public convenience to Pantranco, which submitted the
personal accounts were also paid by the Corporation. Villarama himself admitted that he
sale for approval to the PSC and prayed for provisional authority to operate on the basis of
mingled the corporate funds with his own money.
The foregoing circumstances are strong persuasive evidence showing that Villarama However, the sale between Fernando and the Corporation is valid, such that the rightful
has been too much involved in the affairs of the Corporation to altogether negative the ownership of the disputed certificates still belongs to the plaintiff being the prior purchaser
claim that he was only a part-time general manager. The interference of Villarama in the in good faith and for value thereof. In view of the ancient rule of caveat emptor prevailing
complex affairs of the corporation, and particularly its finances, are much too inconsistent in this jurisdiction, what was acquired by Ferrer in the sheriff's sale was only the right
with the ends and purposes of the Corporation law, which, precisely, seeks to separate which Fernando, judgment debtor, had in the certificates of public convenience on the day
personal responsibilities from corporate undertakings. It is the very essence of of the sale.
incorporation that the acts and conduct of the corporation be carried out in its own
corporate name because it has its own personality.
ADELIO C. CRUZ, complainant vs QUITERIO L. DALISAY, Deputy Sheriff, RTC, Manila,
When the fiction is urged as a means of perpetrating a fraud or an illegal act or as a
respondents
vehicle for the evasion of an existing obligation, the circumvention of statutes, the
achievement or perfection of a monopoly or generally the perpetration of knavery or
crime, the veil with which the law covers and isolates the corporation from the members
Adm. Matter No. R-181-P July 31, 1987
or stockholders who compose it will be lifted to allow for its consideration merely as an
aggregation of individuals.

We hold that the preponderance of evidence have shown that the Villa Rey Transit, Inc. is Fernan, J;
an alter ego of Jose M. Villarama. The rule is that a seller or promisor may not make use
DOCTRINE:
of a corporate entity as a means of evading the obligation of his covenant. Where the
Corporation is substantially the alter ego of the covenantor to the restrictive agreement, it Commercial Law; Corporation. Piercing the veil of corporate entity;A corporation ha a
can be enjoined from competing with the covenantee. personality distinct and separate from its individual stockholders or members; Sheriff
usurped a power that belonged to the court when he chose to “pierce the veil of corporate
We hold the restrictive clause in the contract entered into by the latter and Pantranco is
entity.” –
also enforceable and binding against the said Corporation. As We read the disputed
clause, it is evident from the context thereof that the intention of the parties was to FACTS:
eliminate the seller as a competitor of the buyer for ten years along the lines of operation
In a sworn complaint, Adelio Cruz charged Quiterio Dalisay, Senior Deputy Sheriff of
covered by the certificates of public convenience subject of their transaction.
Manila, with “malfeasance in office, corrupt practices and serious irregularities” allegedly
The rule became well established that if the restraint was limited to "a certain time" and committed as follows:
within "a certain place," such contracts were valid and not "against the benefit of the
state." We find that although it is in the nature of an agreement suppressing competition, 1. Respondent sheriff attached and/or levied the money belonging to complainant
it is, however, merely ancillary or incidental to the main agreement which is that of sale. Cuz when he was not himself the judgment debtor in the final judgment of NLRC
The suppression or restraint is only partial or limited: first, in scope, it refers only to NCR Case sought to be enforced but rather the company known as “Qualitrans
application for TPU by the seller in competition with the lines sold to the buyer; second, in Limousine Service, Inc.” a duly registered corporation, and
duration, it is only for ten (10) years; and third, with respect to situs or territory, the 2. Respondent likewise caused the service of the alias writ of execution UPON
restraint is only along the lines covered by the certificates sold. In view of these limitations, COMPLAINANT who is a resident of Pasay despite knowledge that his territorial
coupled with the consideration of P350,000.00 for just two certificates of public jurisdiction covers Manila only and does not extend to Pasay City.
convenience, and considering, furthermore, that the disputed stipulation is only incidental
In his comments, respondent Dalisay explained that when he garnished complainant’s
to a main agreement, the same is reasonable and it is not harmful nor obnoxious to public
cash deposit at the Philtrust bank, he was merely performing a ministerial duty. While it is
service. The evils of monopoly are farfetched here. There can be no danger of price
true that said writ was addressed to Qualitrans Limousine Service Inc, yet it is also a fact
controls or deterioration of the service because of the close supervision of the Public
that complainant had executed an affidavit before the Pasay City assistant fiscal stating
Service Commission.
that he is the owner/ president of said corporation and because of that declaration, the
counsel for the plaintiff in the labor case advised him to serve notice of garnishment on the
Philtrust bank.

ISSUE:

whether or not the sheriff committed an act that pierced the veil of corporate entity of
Qualitrans?

HELD: YES. G.R. No. 100866 July 14, 1992

We hold that respondent’s actuation in enforcing a judgment against complainant who is REBECCA BOYER-ROXAS and GUILLERMO ROXAS, petitioners,
not the judgment in the case calls for disciplinary action. Considering the ministerial nature vs.
of his duty in enforcing writs of execution, what is incumbent upon him is to ensure that
only that portion of a decision ordained or decreed in the dispositive part should be the HON. COURT OF APPEALS and HEIRS OF EUGENIA V. ROXAS, INC., respondents.
subject of execution. No more, no less. Petition to review the decision and resolution of the Court of Appeals affirming the earlier
The tenor of the NLRC judgment and the implementing writ is clear enough. It directed decision of the RTC-Laguna in the consolidated civil cases involving petitioners and private
Qualitrans Limousine Service Inc to reinstate the discharged employees and pay them respondent Heirs of Eugenia V. Roxas, Inc. (HEIRS), a corporation.
full backwages. Respondent however, chose to “pierce the veil of corporate entity”
usurping a power belonging to the court and assumed improvidently that since the
complainant is the owner/president of Qualitrans Limousine Service, Inc. they are one FACTS: HEIRS filed two separate complaints for recovery of possession with the RTC-
and the same. Laguna against herein petitioners, praying for the ejectment of petitioners from buildings
inside the Hidden Valley Springs Resort, allegedly owned by respondent
It is a well-settled doctrine both in law and in equity that as a legal entity, a corporation corporation. HEIRS alleged that (1) Rebecca Roxas is in possession of two houses built at
has a personality distinct and separate from its individual stockholders or members. The the expense of HEIRS, and that her occupancy on the said houses was only upon the
mere fact that one is president of a corporation does not render the property he owns or tolerance of the corporation; and (2) Guillermo Roxas occupies a house which was built at
possesses the property of the corporation, since the president, as individual, and the the expense of the corporation during the time when Guillermo’s father was still living and
corporation are separate entities. was the general manager of the corporation, and that Guillermo’s possession over the
ACCORDINGLY, we find Respondent Deputy Sheriff Dalisay NEGLIGENT in the enforcement house and lot was only upon the tolerance of the corporation. In their separate answers,
of the writ of execution in NLRC Case no. 8-12389-91; and a fine equivalent to 3 months petitioners traversed the allegations stating that they are heirs of Eugenia V. Roxas, and
salary is hereby imposed with a stern warning that the commission of the same or similar therefore, co-owners of the Hidden Valley Springs Resort; and as co-owners of the
offense in the future will merit a heavier penalty. property, they have the right to stay within its premises.

RTC-Laguna decided in favor of HEIRS. On appeal, the decision of the RTC was
affirmed. Hence, the petition.

ISSUE:Did the CA err when it refused to pierce the veil of corporate fiction over HEIRS and
maintain the petitioners in their possession and/or occupancy of the subject premises
considering that petitioners are owners of aliquot part of the properties of HEIRS?

HELD:NO. The petitioners maintain that their possession of the questioned properties
must be respected in view of their ownership of an aliquot portion of all the properties of
the respondent corporation being stockholders thereof. They propose that the veil of
corporate fiction be pierced, considering the circumstances under which the respondent petitioner Guillermo Roxas managed the corporation. The Board of Directors did not object
corporation was formed. to such an arrangement. The petitioners argue that . . . the authority thus given by
Eufrocino Roxas for the conversion of the recreation hall into a residential house can no
Originally, the questioned properties belonged to Eugenia V. Roxas. After her death, the
longer be questioned by the stockholders of the private respondent and/or its board of
heirs of Eugenia V. Roxas, among them the petitioners herein, decided to form a
directors for they impliedly but no leas explicitly delegated such authority to said Eufrocino
corporation — Heirs of Eugenia V. Roxas, Incorporated (private respondent herein) with
Roxas. (Rollo, p. 12)
the inherited properties as capital of the corporation. The corporation was incorporated on
December 4, 1962 with the primary purpose of engaging in agriculture to develop the
inherited properties. The Articles of Incorporation of the respondent corporation were
Again, we must emphasize that the respondent corporation has a distinct personality
amended in 1971 to allow it to engage in the resort business. Accordingly, the corporation
separate from its members. The corporation transacts its business only through its officers
put up a resort known as Hidden Valley Springs Resort where the questioned properties
or agents. (Western Agro Industrial Corporation v. Court of Appeals, supra). Whatever
are located. These facts, however, do not justify the position taken by the petitioners.
authority these officers or agents may have is derived from the board of directors or other
The respondent is a bona fide corporation. As such, it has a juridical personality of its own governing body unless conferred by the charter of the corporation. An officer's power as
separate from the members composing it. There is no dispute that title over the an agent of the corporation must be sought from the statute, charter, the by-laws or in a
questioned land where the Hidden Valley Springs Resort is located is registered in the delegation of authority to such officer, from the acts of the board of directors, formally
name of the corporation. The records also show that the staff house being occupied by expressed or implied from a habit or custom of doing business. (Vicente v. Geraldez, 52
petitioner Rebecca Boyer-Roxas and the recreation hall which was later on converted into SCRA 210 [1973])
a residential house occupied by petitioner Guillermo Roxas are owned by the respondent
corporation. Regarding properties owned by a corporation, we stated in the case
ofStockholders of F. Guanzon and Sons, Inc. v. Register of Deeds of Manila, (6 SCRA 373 In the present case, the record shows that Eufrocino V. Roxas who then controlled the
[1962]): management of the corporation, being the majority stockholder, consented to the
petitioners' stay within the questioned properties. Specifically, Eufrocino Roxas gave his
xxx xxx xxx
consent to the conversion of the recreation hall to a residential house, now occupied by
. . . Properties registered in the name of the corporation are owned by it as an entity petitioner Guillermo Roxas. The Board of Directors did not object to the actions of
separate and distinct from its members. While shares of stock constitute personal Eufrocino Roxas. The petitioners were allowed to stay within the questioned properties
property, they do not represent property of the corporation. The corporation has property until August 27, 1983, when the Board of Directors approved a Resolution ejecting the
of its own which consists chiefly of real estate (Nelson v. Owen, 113 Ala., 372, 21 So. 75; petitioners.
Morrow v. Gould, 145 Iowa 1, 123 N.W. 743). A share of stock only typifies an aliquot part
of the corporation's property, or the right to share in its proceeds to that extent when
distributed according to law and equity (Hall & Faley v. Alabama Terminal, 173 Ala., 398, 56 We find nothing irregular in the adoption of the Resolution by the Board of Directors. The
So. 235), but its holder is not the owner of any part of the capital of the corporation petitioners' stay within the questioned properties was merely by tolerance of the
(Bradley v. Bauder, 36 Ohio St., 28). Nor is he entitled to the possession of any definite respondent corporation in deference to the wishes of Eufrocino Roxas, who during his
portion of its property or assets (Gottfried V. Miller, 104 U.S., 521; Jones v. Davis, 35 Ohio lifetime, controlled and managed the corporation. Eufrocino Roxas' actions could not have
St., 474). The stockholder is not a co-owner or tenant in common of the corporate property bound the corporation forever. The petitioners have not cited any provision of the
(Harton v. Johnston, 166 Ala., 317, 51 So. 992). (at pp. 375-376) corporation by-laws or any resolution or act of the Board of Directors which authorized
Eufrocino Roxas to allow them to stay within the company premises forever. We rule that
The petitioners point out that their occupancy of the staff house which was later used as
in the absence of any existing contract between the petitioners and the respondent
the residence of Eriberto Roxas, husband of petitioner Rebecca Boyer-Roxas and the
corporation, the corporation may elect to eject the petitioners at any time it wishes for the
recreation hall which was converted into a residential house were with the blessings of
benefit and interest of the respondent corporation.
Eufrocino Roxas, the deceased husband of Eugenia V. Roxas, who was the majority and
controlling stockholder of the corporation. In his lifetime, Eufrocino Roxas together with
Eriberto Roxas, the husband of petitioner Rebecca Boyer-Roxas, and the father of
The petitioners' suggestion that the veil of the corporate fiction should be pierced is The CAruled that petitioner as Senio VP and signatory to the trust receips was criminally
untenable. The separate personality of the corporation may be disregarded only when liable
the corporation is used "as a cloak or cover for fraud or illegality, or to work injustice, or
ISSUES:
where necessary to achieve equity or when necessary for the protection of the
creditors." (Sulong Bayan, Inc. v. Araneta, Inc., 72 SCRA 347 [1976] cited in Tan Boon Bee 1. Whether the certification for non-forum shopping was defective
& Co., Inc., v. Jarencio, supraand Western Agro Industrial Corporation v. Court of
Appeals, supra) The circumstances in the present cases do not fall under any of the
2. Whether petitioner, as Senior VP, could be made liable for violation of the trust
receipts law
enumerated categories.
RULING:
CHING vs. THE SECRETARY OF JUSTICE, et al

G. R. No. 164317 February 6, 2006 1. On the procedural ground the SC ruled that petitioner’s certification for non-
forum shopping was defective because as worded, it cannot even be regarded as
substantial compliance with the procedural requirement
NATURE of the Case: A petition for review on certiorari of the decision of the
Court of Appeals which dismissed petitioner Alfredo Ching’s petition for certiorari,
1. On the issue regarding Ching’s liability:
prohibition and mandamus, essentially allowing an information for violation of the Trust In the case at bar, the transaction between petitioner and respondent bank falls under the
Receipts Law and estafa to be filed against the petitioner trust receipt transactions envisaged in P.D. No. 115. Respondent bank imported the goods
and entrusted the same to PBMI under the trust receipts signed by petitioner, as
FACTS:
entrustee, with the bank as entruster. The agreement was as follows:
Alfredo Ching was was the Senior Vice-President of Philippine Blooming Mills, Inc. (PBMI).
P.D. No. 115 applies to goods used by the entrustee in the operation of its machineries and
In this capacity he applied for the issuance of commercial letters of credit with RCBC. When
equipment. The non-payment of the amount covered by the trust receipts or the non-
granted, petitioner signed 13 trust receipts as surety.
return of the goods covered by the receipts, if not sold or otherwise not disposed of,
When the trust receipts matured, petitioner failed to return the goods to respondent bank, violate the entrustee’s obligation to pay the amount or to return the goods to the
or to return their value amounting to P6.9M despite demands. entruster.
The bank filed a criminal complaint for estafa against petitioner, which was initially by the There are two possible situations in a trust receipt transaction. The first is covered by the
Prosecutor but was reversed on appeal to the Secretary of Justice. provision which refers to money received under the obligation involving the duty to deliver
it (entregarla) to the owner of the merchandise sold. The second is covered by the
The Justice Secretary further stated that the respondent bound himself under the terms of
provision which refers to merchandise received under the obligation to return it
the trust receipts not only as a corporate official of PBMI but also as its surety; hence, he
(devolvera) to the owner.
could be proceeded against in two (2) ways: first, as surety as determined by the Supreme
Court in its decision in Rizal Commercial Banking Corporation v. Court of Appeals; and Thus, failure of the entrustee to turn over the proceeds of the sale of the goods covered by
second, as the corporate official responsible for the offense under P.D. No. 115, via the trust receipts to the entruster or to return said goods if they were not disposed of in
criminal prosecution. Moreover, P.D. No. 115 explicitly allows the prosecution of corporate accordance with the terms of the trust receipt is a crime under P.D. No. 115, without need
officers "without prejudice to the civil liabilities arising from the criminal offense." Thus, of proving intent to defraud.
according to the Justice Secretary, following Rizal Commercial Banking Corporation, the
The Court rules that although petitioner signed the trust receipts merely as Senior Vice-
civil liability imposed is clearly separate and distinct from the criminal liability of the
President of PBMI and had no physical possession of the goods, he cannot avoid
accused under P.D. No. 115.
prosecution for violation of P.D. No. 115.
Petitioner then filed a petition for certiorari, prohibition and mandamus with the CA,
The crime defined in P.D. No. 115 is malum prohibitum but is classified as estafa under
assailing the resolutions of the Secretary of Justice, but the CA dismissed the petition.
paragraph 1(b), Article 315 of the Revised Penal Code, or estafa with abuse of confidence.
Estafa may be committed by a corporation or other juridical entity or by natural persons.
However, the penalty for the crime is imprisonment.
Palting, among others, allegedly prospective investors in the shares of SAN JOSE
Though the entrustee is a corporation, nevertheless, the law specifically makes the PETROLEUM, filed with the SEC an opposition to registration and licensing on the ground
officers, employees or other officers or persons responsible for the offense, without that the tie-up between the issuer, SAN JOSE PETROLEUM, and SAN JOSE OIL, is violative of
prejudice to the civil liabilities of such corporation and/or board of directors, officers, or the Constitution, the Corporation Law, and the Petroleum Act of 1949. SAN JOSE
other officials or employees responsible for the offense. The rationale is that such PETROLEUM claimed that it was a “business enterprise” enjoying parity rights under the
officers or employees are vested with the authority and responsibility to devise means Ordinance appended to the Constitution, which parity right, with respect to mineral
necessary to ensure compliance with the law and, if they fail to do so, are held criminally resources in the Philippines, may be exercised, pursuant to the Laurel-Langley Agreement,
accountable; thus, they have a responsible share in the violations of the law only though the medium of a corporation organized under the laws of the Philippines (SAN
JOSE OIL).
If the crime is committed by a corporation or other juridical entity, the directors, officers,
employees or other officers thereof responsible for the offense shall be charged and
penalized for the crime, precisely because of the nature of the crime and the penalty
ISSUE:
therefor. A corporation cannot be arrested and imprisoned; hence, cannot be penalized for
a crime punishable by imprisonment. However, a corporation may be charged and Is San Jose Petroleum an American business enterprise entitled to parity rights in the
prosecuted for a crime if the imposable penalty is fine. Even if the statute prescribes both Philippines?
fine and imprisonment as penalty, a corporation may be prosecuted and, if found guilty,
HELD:NO.
may be fined.
Firstly — It is not owned or controlled directly by citizens of the United States, because it is
In this case, petitioner signed the trust receipts in question. He cannot, thus, hide behind
owned and controlled by a corporation, the OIL INVESTMENTS, another foreign
the cloak of the separate corporate personality of PBMI. In the words of Chief Justice Earl
(Panamanian) corporation.
Warren, a corporate officer cannot protect himself behind a corporation where he is the
actual, present and efficient actor Secondly — Neither can it be said that it is indirectly owned and controlled by American
citizens through the OIL INVESTMENTS, for this latter corporation is in turn owned and
Palting v. San Jose Petroleum, Inc.
controlled, not by citizens of the United States, but still by two foreign (Venezuelan)
18 SCRA 924 (1966) corporations, the PANTEPEC OIL COMPANY and PANCOASTAL PETROLEUM.

hirdly — Although it is claimed that these two last corporations are owned and controlled
respectively by 12,373 and 9,979 stockholders residing in the different American states,
Petition for review of the order of the Securities and Exchange Commission denying the
there is no showing in the certification furnished by respondent that the stockholders of
opposition to, and instead, granting the registration, and licensing the sale in the
PANCOASTAL or those of them holding the controlling stock, are citizens of the United
Philippines, of shares of the capital stock of the respondent-appellee San Jose Petroleum,
States.
Inc., a corporation organized and existing in the Republic of Panama.
Fourthly — Granting that these individual stockholders are American citizens, it is yet
necessary to establish that the different states of which they are citizens, allow Filipino
FACTS: citizens or corporations or associations owned or controlled by Filipino citizens, to engage
in the exploitation, etc. of the natural resources of these states (see paragraph 3, Article VI
SAN JOSE PETROLEUM is a corporation organized and existing in the Republic of
of the Laurel-Langley Agreement, supra). Respondent has presented no proof to this effect.
Panama. It filed with the SEC a sworn registration statement for the registration and
licensing for sale in the Philippines Voting Trust Certificates representing 2 million shares of Fifthly — But even if the requirements mentioned in the two immediately preceding
its capital stock of a par value of $0.35 a share, at P1 per share. It was alleged that the paragraphs are satisfied, nevertheless to hold that the set-up disclosed in this case, with a
entire proceeds of the sale of said securities will be devoted or used exclusively to finance long chain of intervening foreign corporations, comes within the purview of the Parity
the operations of SAN JOSE OIL COMPANY, a domestic mining corporation, which has 14 Amendment regarding business enterprises indirectly owned or controlled by citizens of
petroleum exploration concessions. the United States, is to unduly stretch and strain the language and intent of the law. For, to
what extent must the word "indirectly" be carried? Must we trace the ownership or ARTICLE VI
control of these various corporations ad infinitum for the purpose of determining whether
1. The disposition, exploitation, development and utilization of all agricultural, timber, and
the American ownership-control-requirement is satisfied? Add to this the admitted fact
mineral lands of the public domain, waters, minerals, coal, petroleum and other mineral
that the shares of stock of the PANTEPEC and PANCOASTAL which are allegedly owned or
oils, all forces and sources of potential energy, and other natural resources of either Party,
controlled directly by citizens of the United States, are traded in the stock exchange in New
and the operation of public utilities, shall, if open to any person, be open to citizens of the
York, and you have a situation where it becomes a practical impossibility to determine at
other Party and to all forms of business enterprise owned or controlled, directly or
any given time, the citizenship of the controlling stock required by the law. In the
indirectly, by citizens of such other Party in the same manner as to and under the same
circumstances, we have to hold that the respondent SAN JOSE PETROLEUM, as presently
conditions imposed upon citizens or corporations or associations owned or controlled by
constituted, is not a business enterprise that is authorized to exercise the parity privileges
citizens of the Party granting the right.
under the Parity Ordinance, the Laurel-Langley Agreement and the Petroleum Law. Its tie-
up with SAN JOSE OIL is, consequently, illegal.

N.B. 2. The rights provided for in Paragraph 1 may be exercised, . . . in the case of citizens of the
United States, with respect to natural resources in the public domain in the Philippines,
Article XIII, Section 1 of the Philippine Constitution provides:
only through the medium of a corporation organized under the laws of the Philippines and
SEC. 1. All agricultural, timber, and mineral lands of the public domain, waters, minerals, at least 60% of the capital stock of which is owned or controlled by citizens of the United
coal, petroleum, and other mineral oils, all forces of potential energy, and other natural States. . . .
resources of the Philippines belong to the State, and their disposition, exploitation,
3. The United States of America reserves the rights of the several States of the United
development, or utilization shall be limited to citizens of the Philippines, or to corporations
States to limit the extent to which citizens or corporations or associations owned or
or associations at least sixty per centum of the capital of which is owned by such citizens,
controlled by citizens of the Philippines may engage in the activities specified in this Article.
subject to any existing right, grant, lease or concession at the time of the inauguration of
The Republic of the Philippines reserves the power to deny any of the rights specified in this
this Government established under this Constitution. . . . (Emphasis supplied)
Article to citizens of the United States who are citizens of States, or to corporations or
In the 1946 Ordinance Appended to the Constitution, this right (to utilize and exploit our associations at least 60% of whose capital stock or capital is owned or controlled by citizens
natural resources) was extended to citizens of the United States, thus: of States, which deny like rights to citizens of the Philippines, or to corporations or
associations which are owned or controlled by citizens of the Philippines. . . . (Emphasis
Notwithstanding the provisions of section one, Article Thirteen, and section eight, Article
supplied.)
Fourteen, of the foregoing Constitution, during the effectivity of the Executive Agreement
entered into by the President of the Philippines with the President of the United States on
Tayag vs. Benguet
the fourth of July, nineteen hundred and forty-six, pursuant to the provisions of
Commonwealth Act Numbered Seven hundred and thirty-three, but in no case to extend An appeal from an order of the Court of First Instance of Manila.
beyond the third of July, nineteen hundred and seventy-four, the disposition, exploitation,
Facts: Idonah Slade Perkins died in New York City in 1960, her estate in the United States
development, and utilization of all agricultural, timber, and mineral lands of the public
which is administered by County Trust Co. adamantly refuses to surrender to her ancillary
domain, waters, minerals, coal, petroleum, and other mineral oils, all forces of potential
administrator in the Philippines, wherein she also left properties specifically two stock
energy, and other natural resources of the Philippines, and the operation of public utilities
certificates from Benguet Consolidated Inc. On motion of Tayag, the ancillary
shall, if open to any person, be open to citizens of the United States, and to all forms of
administrator, the lower court declared that it considered as lost for all purposes in
business enterprises owned or controlled, directly or indirectly, by citizens of the United
connection with the administration and liqiudation of the Philippine estate of Perkins' said
States in the same manner as to, and under the same conditions imposed upon, citizens of
two stock certificates standing in the books of Benguet Consolidated, orderd said
the Philippines or corporations or associations owned or controlled by citizens of the
certificates cancelled; and directed the said corporation to issue new certificates in
Philippines (Emphasis supplied.)
lieu thereof, the same to be delivered by Benguet to the ancillary administrator or probate
In the 1954 Revised Trade Agreement concluded between the United States and the court. Benguet appeals that the said order holding that the same are in existence and in
Philippines, also known as the Laurel-Langley Agreement, embodied in Republic Act the possession of County Trust Co., in New York, therefore not lost. It also invokes a
1355, the following provisions appear: provision of its by laws alleging that "an action regarding ownership of such certificates or
certificates of stock allgedly lost, stolen or destroyed, the issuance of certificates would executory. A writ of execution was issued. It was partially satisfied through garnishment of
await the final decisionby a court regarding the ownership." sums from petitioner's debtor, the Metropolitan Waterworks and Sewerage Authority. An
alias writ was issued to satisfy the balance and the reinstatement of private respondents. It
Issue: Is Benguet Consolidated's claim correct?
could not be served, because the petitioner no longer occupied the premises. Another alias
Held: Benguet does not dispute the authority of authority of the ancillary administrator. It writ was issued, which again could not be served because the employees claimed they
is the duty of the latter to settle the the deceased's estate and satisfy the claims of the were employees of Hydro Pipes Philippines, Inc. and that the properties to be levied upon
creditors. From such premise it would follow that Probate Court could require that the were owned by the said corporation. Private respondents filed a "Motion for Issuance of a
ancillary's right to the two certificates of stocks be respected by Benguet, for the latter is a Break-Open Order," alleging that HPPI and Concept Builders were owned by the same
Philippine corporation owing full allegiance and subject to unrestricted jurisdiction pf local incorporator/stockholders. They also alleged that petitioner temporarily suspended its
courts. Its share of stock could not be considered in any wise as immune from lawful court business operations in order to evade its legal obligations to them and that private
orders. A corporation is an artificial being created by la, it owes its life to the state, its birth respondents were willing to post an indemnity bond to answer for any damages which
being purely dependent on its will. It is inconceivable that it could have more rights and petitioner and HPPI may suffer because of the issuance of the break-open order. The Labor
priveleges of a higher priority than that of its creator. It cannot legitimately refuse to yield Arbiter issued an Order which denied private respondents' motion for break-open order.
obedience to act of its state organs, certainly not excluding the judiciary whenever called On appeal, the NLRC issued the break-open order. Concept Builder’s motion for
upon to do so. reconsideration was denied.

CONCEPT BUILDERS, INC. vs. THE NATIONAL LABOR RELATIONS COMMISSION, (First ISSUE Did the NLRC commit grave abuse of discretion when it issued a "break-open
Division); and Norberto Marabe; Rodolfo Raquel, Cristobal Riego, Manuel Gillego, order" to the sheriff to be enforced against personal property found in the premises of
Palcronio Giducos, Pedro Aboigar, Norberto Comendador, Rogelio Salut, Emilio Garcia, petitioner's sister company?
Jr., Mariano Rio, Paulina Basea, Alfredo Albera, Paquito Salut, Domingo Guarino, Romeo
HELD No
Galve, Dominador Sabina, Felipe Radiana, Gavino Sualibio, Moreno Escares, Ferdinand
Torres, Felipe Basilan, and Ruben Robalos It is a fundamental principle of corporation law that a corporation is an entity separate
and distinct from its stockholders and from other corporations to which it may be
G.R. No. 108734
connected. But, this separate and distinct personality of a corporation is merely a fiction
May 29, 1996 created by law for convenience and to promote justice. So, when the notion of separate
juridical personality is used to defeat public convenience, justify wrong, protect fraud or
HERMOSISIMA, JR., J.
defend crime, or is used as a device to defeat the labor laws, this separate personality of
NATURE Special Civil Action the corporation may be disregarded or the veil of corporate fiction pierced. This is true
likewise when the corporation is merely an adjunct, a business conduit or an alter ego of
FACTSConcept Builders, Inc., a domestic corporation, with principal office at 355 Maysan
another corporation.
Road, Valenzuela, Metro Manila, is engaged in the construction business. Private
respondents were employed by said company as laborers, carpenters and riggers.On The test in determining the applicability of the doctrine of piercing the veil of corporate
November, 1981, private respondents were served individual written notices of fiction is as follows:
termination of employment by petitioner, effective on November 30, 1981, stating that
their contracts of employment had expired and the project in which they were hired had 1. Control, not mere majority or complete stock control, but complete domination,
been completed.However they found that at the time of the termination of their not only of finances but of policy and business practice in respect to the
employment, the project in which they were hired had not yet been finished and transaction attacked so that the corporate entity as to this transaction had at the
completed. Concept Builders had to engage the services of sub-contractors whose workers time no separate mind, will or existence of its own;
performed the functions of private respondents.Private respondents filed a complaint for 2. Such control must have been used by the defendant to commit fraud or wrong,
illegal dismissal, unfair labor practice and non-payment of their legal holiday pay, overtime to perpetuate the violation of a statutory or other positive legal duty or
pay and thirteenth-month pay against petitioner. The Labor Arbiter rendered judgment dishonest and unjust act in contravention of plaintiff's legal rights; and
ordering reinstatement and payment of back wages. The NLRC dismissed the motion for 3. The aforesaid control and breach of duty must proximately cause the injury or
reconsideration filed by Concept Builders, on the ground that it had become final and unjust loss complained of.
Land, Inc. to be listed in its stock market, thus paving the way for the public offering of
PALI's shares
The absence of any one of these elements prevents "piercing the corporate veil." In
applying the "instrumentality" or "alter ego" doctrine, the courts are concerned with FACTS The Puerto Azul Land, Inc. (PALI) sought to offer its shares to the public in order to
reality and not form, with how the corporation operated and the individual defendant's raise funds allegedly to develop its properties and pay its loans with several banking
relationship to that operation. institutions. PALI was issued a Permit to Sell its shares to the public by the SEC. PALI filed
with the PSE an application to list its shares, with supporting documents attached. The
Listing Committee of the PSE, upon a perusal of PALI's application, recommended to the
In this case, the NLRC noted that, while petitioner claimed that it ceased its business PSE's Board of Governors the approval of PALI's listing application.
operations on April 29, 1986, it filed an Information Sheet with the SEC on May 15, 1987,
Before it could act upon PALI's application, the Board of Governors of the PSE received a
stating that its office address is at 355 Maysan Road, Valenzuela, Metro Manila. On the
letter from the heirs of Ferdinand E. Marcos, claiming that the late President Marcos was
other hand, HPPI, the third-party claimant, submitted on the same day, a similar
the legal and beneficial owner of certain properties which PALI claims to be among its
information sheet stating that its office address is at 355 Maysan Road, Valenzuela, Metro
assets and that the Ternate Development Corporation, which is among the stockholders of
Manila.
PALI, likewise appears to have been held and continue to be held in trust by one Rebecco
Both information sheets were filed by the same Virgilio O. Casiño as the corporate Panlilio for then President Marcos and now, effectively for his estate, and requested PALI's
secretary of both corporations. It would also not be amiss to note that both corporations application to be deferred. The Board of Governors of the PSE reached its decision to reject
had the same president, the same board of directors, the same corporate officers, and PALI's application, citing the existence of serious claims, issues and circumstances
substantially the same subscribers. surrounding PALI's ownership over its assets that adversely affect the suitability of listing
PALI's shares in the stock exchange.PALI wrote a letter to the SEC addressed to the then
Clearly, petitioner ceased its business operations in order to evade the payment to private
Acting Chairman, requesting that the SEC, in the exercise of its supervisory and regulatory
respondents of back wages and to bar their reinstatement to their former positions. HPPI
powers over stock exchanges under Section 6(j) of P.D. No. 902-A, review the PSE's action
is obviously a business conduit of petitioner corporation and its emergence was skillfully
on PALI's listing application and institute such measures as are just and proper under the
orchestrated to avoid the financial liability that already attached to petitioner
circumstances. The SEC reversed the PSE’s decision. PSE filed with the Court of Appeals a
corporation.
Petition for Review (with Application for Writ of Preliminary Injunction and Temporary
In view of the failure of the sheriff, in the case at bar, to effect a levy upon the property Restraining Order), assailing the orders of the SEC. The CA dismissed the Petition for
subject of the execution, private respondents had no other recourse but to apply for a Review.
break-open order after the third-party claim of HPPI was dismissed for lack of merit by the
ISSUE
NLRC. This is in consonance with Section 3, Rule VII of the NLRC Manual of Execution of
Judgment. Did the SEC have authority to order the PSE to list the shares of PALI in the stock
exchange?
Hence, the NLRC did not commit any grave abuse of discretion when it affirmed the
break-open order issued by the Labor Arbiter. HELD Section 3 of Presidential Decree 902-A, standing alone, is enough authority to
uphold the SEC's challenged control authority over the petitioner PSE. The SEC's power to
PHILIPPINE STOCK EXCHANGE, INC. vs. THE HONORABLE COURT OF APPEALS, SECURITIES
look into the subject ruling of the PSE, therefore, may be implied from or be considered as
AND EXCHANGE COMMISSION and PUERTO AZUL LAND, INC.
necessary or incidental to the carrying out of the SEC's express power to insure fair dealing
G.R. No. 125469 in securities traded upon a stock exchange or to ensure the fair administration of such
exchange. It is, likewise, observed that the principal function of the SEC is the supervision
October 27, 1997
and control over corporations, partnerships and associations with the end in view that
TORRES, JR., J.: investment in these entities may be encouraged and protected, and their activities for the
promotion of economic development.Thus, it was in the alleged exercise of this authority
NATURE Petition for Review on Certiorari of the resolution of the CA affirming the
that the SEC reversed the decision of the PSE to deny the application for listing in the stock
decision of the Securities and Exchange Commission ordering the PSE to allow Puerto Azul
exchange of the private respondent PALI. The SEC's action was affirmed by the Court of
Appeals. The SEC is the entity with the primary say as to whether or not securities, the SEC may exercise such power only if the PSE's judgment is attended by bad faith. In
including shares of stock of a corporation, may be traded or not in the stock exchange. This Board of Liquidators vs. Kalaw, it was held that bad faith does not simply connote bad
is in line with the SEC's mission to ensure proper compliance with the laws, such as the judgment or negligence. It imports a dishonest purpose or some moral obliquity and
Revised Securities Act and to regulate the sale and disposition of securities in the country. conscious doing of wrong. It means a breach of a known duty through some motive or
This is not to say, however, that the PSE's management prerogatives are under the interest of ill will, partaking of the nature of fraud.
absolute control of the SEC. The PSE is, after all, a corporation authorized by its
It was reasonable for the PSE to exercise its judgment in the manner it deems appropriate
corporate franchise to engage in its proposed and duly approved business.
for its business identity, as long as no rights are trampled upon, and public welfare is
A corporation is but an association of individuals, allowed to transact under an assumed safeguarded.
corporate name, and with a distinct legal personality. In organizing itself as a collective
What is material is that the uncertainty of the properties' ownership and alienability exists,
body, it waives no constitutional immunities and perquisites appropriate to such a body.
and this puts to question the qualification of PALI's public offering.
As to its corporate and management decisions, therefore, the state will generally not
interfere with the same. Questions of policy and of management are left to the honest In sum, the Court finds that the SEC had acted arbitrarily in arrogating unto itself the
decision of the officers and directors of a corporation, and the courts are without authority discretion of approving the application for listing in the PSE of the private respondent
to substitute their judgment for the judgment of the board of directors. The board is the PALI, since this is a matter addressed to the sound discretion of the PSE, a corporation
business manager of the corporation, and so long as it acts in good faith, its orders are not entity, whose business judgments are respected in the absence of bad faith.
reviewable by the courts.

Thus, notwithstanding the regulatory power of the SEC over the PSE, and the resultant
authority to reverse the PSE's decision in matters of application for listing in the market,

Das könnte Ihnen auch gefallen